27.08.2013 Aufrufe

zurück

zurück

zurück

MEHR ANZEIGEN
WENIGER ANZEIGEN

Sie wollen auch ein ePaper? Erhöhen Sie die Reichweite Ihrer Titel.

YUMPU macht aus Druck-PDFs automatisch weboptimierte ePaper, die Google liebt.

Fachhochschule Köln - Fachbereich Nachrichtentechnik<br />

Institut für Niederfrequenztechnik und Technische Akustik<br />

Technische Akustik<br />

Prof. Dr.-Ing. J. Leichsenring<br />

Klausuren und Lösungen<br />

Diese Klausursammlung enthält mindestens fünf der<br />

neuesten Klausuren. Neben den Aufgaben sind zu Ihrer<br />

Kontrolle die Lösungen beigefügt.<br />

Diese Lösungen enthalten bewußt nicht die Lösungswege,<br />

obwohl diese für die Bewertung Ihrer Klausur<br />

besonders wichtig sind.<br />

Achten Sie also darauf, daß Sie nie einfach Ergebnisse<br />

übernehmen, sondern immer vom Ansatz bis zum<br />

Ergebnis einen nachvollziehbaren Lösungsweg<br />

dokumentieren.<br />

Wenn Sie Fragen haben, kommen Sie bitte in meine Sprechstunden<br />

oder senden Sie mir eine e-mail:<br />

Prof. Dr. J. Leichsenring: juergen.leichsenring@fh-koeln.de


FACHHOCHSCHULE KÖLN FACHBEREICH NACHRICHTENTECHNIK<br />

TECHNISCHE AKUSTIK<br />

KLAUSUR ANFANG SS 2001/02 AM 12. MÄRZ 2002<br />

Prof. Dr.-Ing. J.L Leichsenring<br />

d<br />

Anregung Messpunkt<br />

D<br />

reflexionsfreier<br />

Abschluss<br />

c:/Eigene Dateien/Lotus/Klausuren/TA/MAR_02/TAK_MAR_02_net.lwp<br />

d<br />

Aufgabe 1: (25 Punkte)<br />

Ein Tieftonlautsprecher im geschlossenen Gehäuse mit einem<br />

Membrandurchmesser D = 10cm ist für einen maximalen<br />

Membranhub xss= 8 mm (Spitze- Spitzewert)<br />

ausgelegt. Er soll im diffusen Schallfeld eines Raums mit<br />

V = 40 m und einem Hallradius bei<br />

3 rh = 1m f = 100Hz<br />

einen Schallpegel L = 100 dB erzeugen.<br />

Sie sollen prüfen, ob der Lautsprecher die Forderung<br />

erfüllen kann.<br />

a) Es gibt zwei prinzipiell verschiedene Lösungswege.<br />

Beschreiben Sie beide Wege. (5P)<br />

b) Führen Sie einen der beiden Lösungswege aus.<br />

(20P)<br />

Aufgabe 2: (10 Punkte)<br />

Ein Stahlstab ( E = 2 $ 10 , ) mit<br />

11 N/m2 ✯ = 7800 kg/m3 der Breite b = 10 mm und der Dicke h = 1 mm wird mit<br />

Biegewellen angeregt. Die Anregung enthält die Frequenzen<br />

f = 74 Hz und f = 222 Hz.<br />

Berechnen Sie den Laufzeitunterschied der beiden<br />

Wellen im Abstand x = 100 mm von der Anregungsstelle.<br />

Aufgabe 3: (15 Punkte)<br />

Für ein Kondensatormikrofon ist gegeben:<br />

Membrandurchmesser D = 2cm<br />

Elektrodenabstand d = 30 ✙m<br />

Vorspannung U = 120 V<br />

elektr. Feldkonstante ✒0 = 8, 85 As/(Vm)<br />

mech. Nachgiebigkeit n = 0, 7 ✙m/N<br />

a) Bestimmen Sie den Wandlerfaktor N (5P)<br />

b) Bestimmen Sie den Übertragungsfaktor TpU (5P)<br />

c) Welche Spannung gibt das Mikrofon ab, wenn es sich<br />

in einem Schallfeld mit den Schalldruckpegel<br />

L = 80dB befindet? (5P)


TECHNISCHE AKUSTIK 12. MÄRZ 2002 Seite 4<br />

————————————————————————————————————————<br />

FRAGENTEIL<br />

1<br />

2<br />

3<br />

4<br />

5<br />

6<br />

7<br />

8<br />

9<br />

10<br />

11<br />

12<br />

13<br />

Diphone nennt man in der Sprachverarbeitung die Kombination<br />

± zweier Worte ± zweier Formanten ± zweier Laute<br />

Der Übertragungsfaktor eines dynamischen Tieftonlautsprechers hängt in seinem<br />

Übertragungsbereich ab vom Verhältnis der<br />

± Massen von Membran und Schallfeldbelastung<br />

± Induktivität und Widerstand der Schwingspule<br />

± Wirk- und Blindanteile der Schallfeldbelastung<br />

Die Einheit ist die Einheit der<br />

Ns/m 5<br />

± akustischen Impedanz ± Dichte ± Energiedichte<br />

Welches Spektrum haben Geräusche?<br />

± ein Linienspektrum<br />

± ein kontinuierliches Spektrum<br />

± gar keins, weil sie nicht periodisch sind<br />

Ein Kondensatormikrofon muss an seinen Klemmen<br />

± sehr hochohmig ± sehr niederohmig ± induktiv<br />

abgeschlossen werden<br />

Unter einem schallweichen Abschluss versteht man einen Abschluss,<br />

± dessen spez. Impedanz klein gegen die des 1. Mediums ist<br />

± dessen spez. Impedanz groß gegen die des 1. Mediums ist<br />

± an dem keine Reflexion auftritt<br />

In einer mech. Ersatzschaltung hat<br />

± die Feder ± die Masse ± der Widerstand nur einen Anschluß<br />

Die Schallgeschwindigkeit von Transversalwellen in einem unendlich ausgedehnten festen<br />

Körper hängt<br />

± linear ± quadratisch ± nicht von der Frequenz ab<br />

Bei zwei parallelgeschalteten Massen m1= m2= 10g findet man<br />

± die Gesamtmasse 20g<br />

± die Gesamtmasse 5g<br />

± dass die Parallelschaltung nicht möglich ist<br />

Die Stelle im Vokaltrakt, wo ein Laut entsteht, bezeichnet man als<br />

± Formant ± Artikulationsort ± Stimmritze<br />

Der Reflexionsfaktor an einer Grenzfläche ist definiert als das Verhältnis der Schalldrücke des<br />

± absorbierten ± durchtretenden ± reflektierten<br />

und des hinlaufenden Schalldrucks<br />

In einer stehenden Welle verhalten sich die Schalldrücke in den Minima und Maxima wie 3:5,<br />

der Reflexionsfaktor ist dann<br />

± 0,25 ± 0,6 ± 0,15<br />

Zwei in Reihe geschaltete akustische Massen kann man ersetzen durch eine akustische<br />

Gesamtmasse mit<br />

± Mak1 = Mak1 + Mak2<br />

± 1/Mak = 1/Mak1 + 1/Mak2<br />

± die Zusammenfassung ist nicht möglich


TECHNISCHE AKUSTIK 12. MÄRZ 2002 Seite 5<br />

————————————————————————————————————————<br />

FRAGENTEIL<br />

14<br />

15<br />

16<br />

17<br />

18<br />

19<br />

20<br />

21<br />

22<br />

23<br />

24<br />

25<br />

Der Hallradius ist<br />

± die Reichweite einer elektroakustischen Anlage<br />

± der Abstand von einem Kugelstrahler nullter Ordnung, bei dem sein direktes Schallfeld<br />

gleich dem stationären Schallfeld ist<br />

± der Abstand aller Punkte gleicher Nachhallzeit vom Mittelpunkt eines Kugelstrahlers nullter<br />

Ordnung<br />

Baut man einen Tieftonlautsprecher in ein geschlossenes Gehäuse ein, so wird seine<br />

Resonanzfrequenz<br />

± sinken ± gleich bleiben ± steigen<br />

Die Einheit der Nachgiebigkeit ist<br />

± 1 N/m ± 1 m/N ± 1 N/m 2<br />

Bei Dehnwellen auf schlanken Stäben<br />

± tritt Querkontraktion auf<br />

± bleibt die Querschnittsfläche konstant<br />

± treten Drehmomente auf<br />

Eine Nierencharakteristik erhält man durch Überlagerung der Schallfelder<br />

± zweier Kugelstrahler nullter Ordnung<br />

± zweier Kugelstrahler erster Ordnung<br />

± je eines Kugelstrahlers erster und nullter Ordnung<br />

Im empfindlichsten Frequenzbereich liegt die Hörschwelle bei etwa<br />

± ± ±<br />

5 $ 10 −8 Pa 2 $ 10 −5 Pa 10 −12 Pa<br />

Kolbenstrahler nullter Ordnung kann man zur Berechnung<br />

± in jedem Fall durch Kugelstrahler ersetzen<br />

± für hohe Frequenzen durch Kugelstrahler nullter Ordnung nähern<br />

± für tiefe Frequenzen durch Kugelstrahler nullter Ordnung nähern<br />

unter "rosa Rauschen" versteht man ein breitbandiges Rauschen, dessen Intensitätsdichtepegel<br />

je Verdopplung der Frequenz<br />

± um 3 dB ansteigt ± um 3 dB abfällt ± konstant bleibt<br />

In einem Kugelschallfeld nullter Ordnung sind Druck und Schnelle<br />

± immer in Phase<br />

± immer um 90 phasenverschoben<br />

o<br />

± in Phase für kr p 1<br />

Unter Residuum-Effekt versteht man<br />

± die Resthörfähigkeit bei stark Schwerhörigen<br />

± die Verzerrungen des Gehörs<br />

± die gehörte Tonhöhe bei fehlendem Grundton<br />

Eine Schallwelle wird vollständig absorbiert, wenn die Schallkennimpedanz Z0 des Mediums,<br />

auf das der Schall trifft<br />

± angepasst ist ± schallweich ist ± schallhart ist<br />

Ein Biegeschwinger ist<br />

± ein Federpendel<br />

± eine Form eines piezoelektrischen Wandlers<br />

± die Membran eines Kondensatormikrofons


M 2<br />

Aufgabe 3: (15 Punkte)<br />

FACHHOCHSCHULE KÖLN FACHBEREICH NACHRICHTENTECHNIK<br />

TECHNISCHE AKUSTIK<br />

KLAUSUR ANFANG WS 2001/02 AM 25. SEPTEMBER 2001<br />

Prof. Dr.-Ing. J.L Leichsenring<br />

Diese Klausur gliedert sich in zwei Teile:<br />

Der Aufgabenteil enthält drei Aufgaben, die mit insgesamt 50 Punkten bewertet werden.<br />

Der Fragenteil enthält 25 Fragen, die mit jeweils zwei Punkten bewertet werden. Von jeweils drei<br />

Antworten ist nur eine richtig. Um Manipulationen auszuschließen, wird innerhalb des Fragenteils von der<br />

hier erreichten Punktzahl für jede falsch angekreuzte Antwort ein Punkt abgezogen. Statistisches<br />

Ankreuzen ergibt damit null Punkte. Es darf nur jeweils eine Antwort angekreuzt werden.<br />

Die Klausur ist mit 50 Punkten bestanden.<br />

d<br />

d 1<br />

u 1<br />

u 2<br />

γ<br />

M 1<br />

1<br />

e j45o<br />

ebene<br />

Welle<br />

unendliche<br />

Wand<br />

l<br />

d 2<br />

+<br />

u a<br />

Aufgabe 1: (20 Punkte)<br />

Zwei gleiche Mikrofone mit Kugelcharakteristik sind in<br />

einem Abstand d = λ/4 im ebenen Schallfeld<br />

angeordnet. Die Mikrofonspannungen werden um den<br />

gleichen Faktor verstärkt, zuätzlich wird die Phase von<br />

u2 um 45° gegen u1 gedreht.<br />

a) Entwickeln Sie eine Gleichung für den Richtfaktor<br />

Γ dieser Anordnung.<br />

b) Skizzieren Sie das Richtdiagramm, indem Sie Γ in<br />

Abhängigkeit von γ in Schritten von 20° berechnen.<br />

Nutzen Sie Symmetrieeigenschaften aus!<br />

Aufgabe 2: (15 Punkte)<br />

Für einen Hochton-Kalottenlautsprecher mit eimem<br />

Membrandurchmesser d1 = 3cm soll ein Exponentialtrichter<br />

entworfen werden.<br />

a) Berechnen Sie das Wuchsmaß g des Tichters für<br />

eine untere Grenzfrequenz fgu = 2 kHz.<br />

b) Bestimmen sie den Öffnungsdurchmesser d2,<br />

wenn bei f = 2 kHz k·räq = 1 gefordert ist.<br />

c) Bestimmen Sie die Länge l des Trichters.<br />

Zwei Geräusche erzeugen am Ohr einzeln dargeboten jeweils einen Lautstärkepegel Ls = 70phon.<br />

a) Welche gesamte Lautheit und welcher gesamte Lautstärkepegel ergibt sich, wenn beide<br />

Geräusche in der selben Frequenzgruppe liegen?<br />

b) Welche gesamte Lautheit und welcher gesamte Lautstärkepegel ergibt sich, wenn beide<br />

Geräusche in verschiedenen Frequenzgruppen liegen?


TECHNISCHE AKUSTIK 25. SEPTEMBER 2001 Seite 2<br />

————————————————————————————————————————<br />

FRAGENTEIL<br />

1<br />

2<br />

3<br />

4<br />

5<br />

6<br />

7<br />

8<br />

9<br />

10<br />

11<br />

12<br />

13<br />

Ein Druckgradientenmikrofon ist ein Mikrofon<br />

± mit Achtercharakteristik<br />

± mit Nierencharakteristik<br />

± mit Druckausgleich<br />

Bei einer Strahlerzeile aus idealen Kugelstrahlern nullter Ordnung ist die Richtcharakteristik in<br />

einer Ebene senkrecht zur Zeile<br />

± winkelunabhängig<br />

± die Funktion Γ = |(sin nx)/(n sin x)|<br />

± unbestimmt<br />

Schall breite sich in einem Rohr mit Querschnittssprung aus. Unmittelbar vor und hinter dem<br />

Querschnittssprung sind die<br />

± Schnellen ± Drücke ± Reflexionsfaktoren gleich<br />

Die Verständlichkeit und Natürlichkeit einer synthetischen Sprache ist dann hoch, wenn sie aus<br />

± besonders kleinen Bausteinen wie Phonemen generiert wird,<br />

± möglichst großen Bausteinen wie Silben generiert wird.<br />

± Die Größe der Bausteine spielt keine Rolle.<br />

Um am Ausgang eines Terzfiltersatzes in jedem Bereich den gleichen Effektivwert der Spannung<br />

zu messen, muss man am Eingang<br />

± weißes ± "rosa" ± oktavgefiltertes Rauschen angelegen<br />

Die Phasengeschwindigkeit des Schalls ist frequenzabhängig bei<br />

± Dichtewellen ± Biegewellen ± Transversalwellen<br />

Die Einheit der Nachgiebigkeit ist<br />

± 1 N/m ± 1 m/N ± 1 N/m2<br />

Die Stelle im Vokaltrakt, wo ein Laut entsteht, bezeichnet man als<br />

± Stimmritze ± Formant ± Artikulationsort<br />

Der Absorptionsgrad an einer Grenzfläche ist definiert als Verhältnis der Schallenergiedichten<br />

des<br />

± hinlaufenden und reflektierten<br />

± reflektierten und hinlaufenden<br />

± absorbierten und hinlaufenden Schalls<br />

Der Absorptionsexponent ist<br />

± besonders hochwertiges Schallschluckmaterial<br />

± ein anderer Name für den Schallabsorptionsgrad<br />

± ein Teil der Nachhallgleichung von Waetzmann, Schuster und Eyring<br />

Die Gesamtlautstärke zweier Geräusche, die in unterschiedlichen Frequenzgruppen liegen, kann<br />

man berechnen aus der Summe der<br />

± Lautheiten ± Lautstärken ± Intensitäten<br />

Die von uns benutzte Wellengleichung für Schallwellen in Luft gilt für<br />

± strömende Luft ohne Leistungsverluste<br />

± ruhende Luft mit Leistungsverlusten<br />

± ruhende Luft ohne Leistungsverluste<br />

Mech. Ersatzschaltungen sind zweckmäßig zur Beschreibung von<br />

± akustischen Filterschaltungen<br />

± Impedanzen im Schallfeld<br />

± mechanischen Strukturen


TECHNISCHE AKUSTIK 25. SEPTEMBER 2001 Seite 3<br />

————————————————————————————————————————<br />

FRAGENTEIL<br />

14<br />

15<br />

16<br />

17<br />

18<br />

19<br />

20<br />

21<br />

22<br />

23<br />

24<br />

25<br />

Eine Schallquelle gibt eine konstante akustische Leistung ab. Der Pegel im stationären Schallfeld<br />

des Raums<br />

± steigt bei zunehmender Nachhallzeit<br />

± fällt bei zunehmender Nachhallzeit<br />

± ist von der Nachhallzeit unabhängig<br />

Die Membranmasse eines dynamischen Tieftonlautsprechers liegt in der Größenordnung<br />

± 100 mg ± 10 g ± 1 kg<br />

Der Longitudinalwellenmodul L<br />

± ist gleich dem Elastizitätsmodul E<br />

± bestimmt die Phasengeschwindigkeit einer Dichtewelle<br />

± bestimmt die Phasengeschwindigkeit einer Dehnwelle<br />

In Rohren mit gleichbleibendem Querschnitt können bei tiefen Frequenzen auftreten<br />

± Kugelwellen ± Zylinderwellen ± ebene Wellen<br />

Einer mech. Parallelschaltung aus Masse und Widerstand wird eine elektrische Parallelschaltung<br />

aus Widerstand und Induktivität gegenübergestellt. Die Analogie ist:<br />

± dual ± schaltungstreu ± falsch<br />

Der Übertragungsfaktor eines dynamischen Tieftonlautsprechers hängt in seinem<br />

Übertragungsbereich ab vom Verhältnis der<br />

± Massen von Membran und Schallfeldbelastung<br />

± Induktivität und Widerstand der Schwingspule<br />

± Wirk- und Blindanteile der Schallfeldbelastung<br />

Bei Messungen an einem Kolbenstrahler muss man einen bestimmten Mindestabstand einhalten,<br />

da im Nahfeld<br />

± Interferenzen auftreten<br />

± Druck und Schnelle nicht in Phase sind<br />

± der hohe Schalldruck das Messmikrofon zerstören könnte<br />

Unter Mithörschwelle versteht man<br />

± einen gerade hörbaren Pegel<br />

± einen gerade hörbaren Pegel bei Anwesenheit eines anderen<br />

± die Verdeckung eines Pegels durch einen anderen<br />

Will man ein Rohr so abschließen, dass eine Schallwelle dort vollständig absorbiert wird, so muss<br />

der Abschluss<br />

± gleich dem Wellenwiderstand des Mediums im Rohr<br />

± schallweich<br />

± schallhart sein<br />

Bei einem Kondensatormikrofon ist die abgegebene Spannung proportional zur<br />

± Schnelle ± Beschleunigung ± Auslenkung der Membran<br />

Die Schallfeldbelastung beim Kugelstrahler erster Ordnung kann man bei sehr tiefen Frequenzen<br />

nähern durch<br />

± einen mechanischen Widerstand ± eine Masse ± eine Feder<br />

Die Einführung eines Schnelle- bzw. Geschwindigkeitspotentials<br />

± ermöglicht eine übersichtliche Form der Wellengleichung<br />

± gestattet die Berechnung von Lautsprecherweichen<br />

± ermöglicht eine Beschreibung des Hörvorgangs


Diese Klausur gliedert sich in zwei Teile:<br />

Der Aufgabenteil enthält drei Aufgaben, die mit insgesamt 50 Punkten bewertet werden.<br />

Der Fragenteil enthält 25 Fragen, die mit jeweils zwei Punkten bewertet werden. Von jeweils drei Antworten<br />

ist nur eine richtig. Um Manipulationen auszuschließen, wird innerhalb des Fragenteils von der hier<br />

erreichten Punktzahl für jede falsch angekreuzte Antwort ein Punkt abgezogen. Statistisches Ankreuzen<br />

ergibt damit null Punkte. Es darf nur jeweils eine Antwort angekreuzt werden.<br />

Die Klausur ist mit 50 Punkten bestanden.<br />

Aufgabe 1: (20 Punkte)<br />

Aufgabe 2: (10 Punkte)<br />

Aufgabe 3: (20 Punkte)<br />

FACHHOCHSCHULE KÖLN FACHBEREICH NACHRICHTENTECHNIK<br />

TECHNISCHE AKUSTIK<br />

KLAUSUR ENDE SS 2001 AM 06. JULI 2001<br />

Prof. Dr.-Ing. J.L Leichsenring<br />

Ein dynamisches Mikrofon soll im Übertragngsbereich einen Leerlauf-Druckübertragungsfaktor<br />

T pU = 0,2 mV/Pa aufweisen. Von den Mikrofondaten sind bekannt:<br />

die Membranfläche SM = 3cm 2<br />

die Membranmasse m M = 0,1 g<br />

die mech. Kreisgüte QM = 0,1<br />

die Resonanzfrequenz f0 = 1400 Hz<br />

die Luftspanltinduktion B = 1Vs/m 2<br />

a) Bestimmen Sie die Länge der Wicklung der Schwingspule für den geforderten<br />

Druckübertragungsfaktor.<br />

b) Bestimmen sie die obere und die untere Grenzfrequenz des Übertragungsbereichs.<br />

In einem Raum mit den Abmessungen 4m$7 m$2, 7 m und einer Nachhallzeit T = 0,6 s wird<br />

Oktavrauschen bei 1 kHz von einem Lautsprecher mit einem elektroakustischen Wirkungsgrad<br />

✔ = 0,7 % wiedergegeben.<br />

Welchen Schallpegel mißt man im diffusen Feld, wenn man dem Lautsprecher 1 W elektrische<br />

Leistung zuführt?<br />

Eine ebene Welle trifft auf einen Abschluß mit Zspez =(1 + 3j) $ Z 0 senkrecht auf.<br />

a) Wie groß ist die Welligkeit m der stehenden Welle vor der Wand?<br />

b) Berechnen Sie den Abstand des ersten Minimums vor der Wand (c = 345 m/s, f = 800 Hz).


TECHNISCHE AKUSTIK 6. JULI 2001<br />

————————————————————————————————————————<br />

FRAGENTEIL<br />

1<br />

2<br />

3<br />

4<br />

5<br />

6<br />

7<br />

8<br />

9<br />

10<br />

11<br />

12<br />

Das Gesetz der ersten Wellenfront besagt, daß<br />

± der Schall von der nächstliegenden Quelle zuerst eintrifft<br />

± zwei Wellenfronten ein Echo ergeben<br />

± unser Gehör eine Quelle nach der zuerst eintreffenden Wellenfront ortet<br />

Akustische Schaltelemente werden nur verwendet, wenn<br />

± die Abmessungen klein gegen die Wellenlänge sind<br />

± die Schalldruckamplituden klein bleiben<br />

± keine Verluste auftreten<br />

Der Quotient Kraft/Schnelle beschreibt eine<br />

± spezifische Schallfeldimpedanz<br />

± akustische Impedanz<br />

± mechanische Impedanz<br />

In einer mechanischen "Schaltung" gilt die Knotenregel für<br />

± die Schnellen ± die Kräfte ± die Ausschläge<br />

Bei der Unterscheidung von Vokalen kommt es wesentlich auf<br />

± die Tonhöhe der Stimme an<br />

± die Hüllkurve des Leistungsdichtespektrums an<br />

± eine streng periodische Anregung der Stimmbänder an<br />

Unter Schallschnelle versteht man die<br />

± Geschwindigkeit eines Materieteilchens in der Schallwelle<br />

± Ausbreitungsgeschwindigkeit des Schalls<br />

± Erdbeschleunigung<br />

Betreibt man einen Kugelstrahler nullter Ordnung und einen Kugelstrahler erster Ordnung<br />

im Abstand d


TECHNISCHE AKUSTIK 6. JULI 2001<br />

————————————————————————————————————————<br />

FRAGENTEIL<br />

13<br />

14<br />

15<br />

16<br />

17<br />

18<br />

19<br />

20<br />

21<br />

22<br />

23<br />

24<br />

25<br />

Der Absorptionsexponent ist<br />

± besonders hochwertiges Schallschluckmaterial<br />

± ein anderer Name für den Schallabsorptionsgrad<br />

± ein Teil der Nachhallgleichung von Waetzmann, Schuster und Eyring<br />

In einer stehenden Welle bewegt sich das erste Minimum vor dem Abschluß<br />

± auf den Abschluß zu ± auf die Schallquelle zu ± gar nicht, es ist ortsfest<br />

Die Zahl der Phoneme der Deutschen Sprache beträgt etwa<br />

± 10 ± 50 ± 500<br />

Von "Nahbesprechungseffekt" spricht man, wenn<br />

± bei einem Richtmikrofon die tiefen Frequenzen überbetont werden<br />

± der Zuhörer sich vor Feuchtigkeit schützen muß<br />

± ein Popstar aus Versehen das Mikrofon verschluckt<br />

Bei einem Exponentialtrichter mit unendlicher Länge wird unterhalb seiner<br />

Grenzfrequenz<br />

± nur sehr wenig ± sehr gut ± keine Wirkleistung übertragen<br />

Unter Querkontraktion einer stabförmigen Probe versteht man<br />

± Zusammenziehung bei Abkühlung<br />

± Querschittsreduktion bei Dehnung<br />

± Zerstörung unter Belastung<br />

Ein Mikrofon, das als Schnelleempfänger arbeitet, hat die Richtcharakteristik einer<br />

± Kugel ± Niere ± Acht<br />

Kolbenstrahler kann man zur Berechnung<br />

± in jedem Fall durch Kugelstrahler ersetzen<br />

± nur durch Kugelstrahler nullter Ordnung ersetzen<br />

± für tiefe Frequenzen durch einen entsprechenden Kugelstrahler nähern<br />

unter "rosa Rauschen" versteht man ein breitbandiges Rauschen, dessen<br />

Intensitätsdichtepegel je Verdopplung der Frequenz<br />

± um 3 dB ansteigt ± um 3 dB abfällt ± konstant bleibt<br />

Bei einem Kondensatormikrofon im Betrieb mit konstanter Ladung ist die<br />

Ausgangsspannung proportional zu<br />

± der Membranauslenkung x<br />

± der Membranschnelle v<br />

± dem Quadrat der Membranauslenkung x 2<br />

Unter Residuum-Effekt versteht man<br />

± die Resthörfähigkeit bei stark Schwerhörigen<br />

± die Verzerrungen des Gehörs<br />

± die gehörte Tonhöhe bei fehlendem Grundton<br />

Die Übertragungsfunktion eines Tieftonlautsprechers im geschlossenen Gehäuse ist die<br />

eines Hochpasses<br />

± erster ± zweiter ± dritter Ordnung<br />

Der Absorptionsgrad an einer Grenzfläche ist definiert als das Verhältnis der<br />

± Schalldrücke des hinlaufenden und reflektierten Schalls<br />

± Schalldrücke des absorbierten und hinlaufenden Schalls<br />

± Schallenergiedichten des absorbierten und hinlaufenden Schalls


fmOktav/Hz<br />

250<br />

500<br />

1000<br />

2000<br />

4000<br />

8000<br />

FACHHOCHSCHULE KÖLN FACHBEREICH NACHRICHTENTECHNIK<br />

TECHNISCHE AKUSTIK<br />

KLAUSUR ENDE WS 2000/2001 AM 13. FEBRUAR 2001<br />

Prof. Dr.-Ing. J.L. Leichsenring<br />

Aufgabe 1: (25 Punkte) Gegeben ist ein Breitbandlautsprecher mit folgeden Daten:<br />

Membrandurchmesser: D = 10 cm<br />

Membranmasse: mM = 10 g<br />

Resonanzfrequenz: f0 = 40 Hz<br />

Dieser Lautsprecher wird in ein geschlossenes Gehäuse mit dem Volumen V = 10Liter eingebaut.<br />

Die resultierende Kreisgüte erreicht nach dem Einbau beim Anschluß an den Verstärker den Wert<br />

QT = 1,2.<br />

a) (10 Punkte) Welchen Wert erreicht die Kennfrequenz des Lautsprechers f0T nach dem Einbau?<br />

b) (5 Punkte) Wie groß ist das Äquivalenzvolumen VAS des Lautsprchers?<br />

c) (10 Punkte) Wo liegt die untere Grenzfrequenz der Übertragungsfunktion?<br />

T/s<br />

1,4<br />

1,3<br />

1,0<br />

0,7<br />

0,5<br />

0,6<br />

LOktav/dB<br />

66<br />

83<br />

72<br />

76<br />

60<br />

55<br />

A<br />

Diese Klausur gliedert sich in zwei Teile:<br />

Der Aufgabenteil enthält zwei Aufgaben, die mit insgesamt 50 Punkten bewertet werden.<br />

Der Fragenteil enthält 25 Fragen, die mit jeweils zwei Punkten bewertet werden. Von jeweils drei Antworten<br />

ist nur eine richtig. Um Manipulationen auszuschließen, wird innerhalb des Fragenteils von der hier<br />

erreichten Punktzahl für jede falsch angekreuzte Antwort ein Punkt abgezogen. Statistisches Ankreuzen<br />

ergibt damit null Punkte. Es darf nur jeweils eine Antwort angekreuzt werden.<br />

Die Klausur ist mit 50 Punkten bestanden.<br />

Aufgabe 2: (25 Punkte)<br />

In einem Raum mit den Abmessungen 4,3m$8m$2,4m wurden<br />

die in der Tabelle eingetragenen Nachhallzeiten gemessen.<br />

Beim Betrieb einer Handbohrmaschine ergibt eine Oktavpegelmessung<br />

im diffusen Feld die gegebenen Pegelwerte.<br />

Bestimmen sie die gesamte von der Bohrmaschine abgegebene<br />

akustische Leistung.


TECHNISCHE AKUSTIK 12. FEBRUAR 2001<br />

————————————————————————————————————————<br />

FRAGENTEIL<br />

1<br />

2<br />

3<br />

4<br />

5<br />

6<br />

7<br />

8<br />

9<br />

10<br />

11<br />

12<br />

13<br />

14<br />

Stellt man den zeitlichen Verlauf eines Sprachspektrums als Graustufen in Abhängigkeit<br />

von Zeit und Frequenz dar, so erhält man<br />

± ein Spektrogramm ± eine Formantkurve ± ein Cepstrum<br />

Ein Bändchenmikrofon ist ein Mikrofon<br />

± in Miniaturausführung<br />

± mit einer bändchenförmigen leitenden Membran<br />

± zur Übertragung eines schmalen Frequenzbandes<br />

Ein Abgasschalldämpfer bei Verbrennungsmotoren enthält einen akustischen<br />

± Resonator ± Tiefpaß ± Hochpaß<br />

Die zeitliche Dichte der Rückwürfe in einem Raum ist bei impulsförmiger Anregung<br />

proportional<br />

± zur Zeit ± zum Quadrat der Zeit ± zur Anregungsfrequenz<br />

Der Ausdruck Diphone bezeichnet in der Sprachverarbeitung die Kombination<br />

± zweier Allophone ± zweier Worte ± zweier Formanten<br />

Die in einem einseitig geschlossenen Rohr auftretenden Eigenfrequenzen sind alle<br />

± ganzzahligen ± ungeradzahligen ± geradzahligen<br />

Vielfachen der tiefsten Eigenfrequenz<br />

Zwei parallelgeschaltete Federn mit den Nachgiebigkeiten n1=3m/N und n2=2m/N haben<br />

die Gesamtnachgiebigkeit<br />

± 5,00 m/N ± 0,67 m/N ± 1,2 m/N<br />

Die Querkontraktionszahl µ gibt den Zusammenhang zwischen<br />

± Dehnung und Stauchung<br />

± Querdehnung und Stauchung<br />

± Querdehnung und Querkontraktion an<br />

Mech. Ersatzschaltungen sind zweckmäßig zur Beschreibung von<br />

± akustischen Filterschaltungen<br />

± Impedanzen im Schallfeld<br />

± mechanischen Strukturen<br />

Die menschliche Stimme erzeugt Schalleistungen in der Größenordnung von<br />

± 1 W ± 1 mW ± 1 µW<br />

An einem Grenzübergang zwischen zwei Medien wird die gesamte Schallenergie<br />

reflektiert, wenn das zweite Medium eine Schallkennimpedanz hat, die<br />

± Z02 = 0 ± Z01 < Z02 ± Z01 > Z02 ist.<br />

Die Schallenergiedichte ist definiert als<br />

± Schalleistung/Fläche ± Schallenergie/Volumen ± Schallleistung/Volumen<br />

Der Realteil der einen Kugelstrahler erster Ordnung belastenden Impedanz steigt bei<br />

tiefen Frequenzen proportional zu<br />

± f 2 ± f 3 ± f 4<br />

Die Sabine'sche Nachhallformel gilt<br />

± immer ± nur für kleine α ± nur für hohe Frequenzen


TECHNISCHE AKUSTIK 12. FEBRUAR 2001<br />

————————————————————————————————————————<br />

FRAGENTEIL<br />

15<br />

16<br />

17<br />

18<br />

19<br />

20<br />

21<br />

22<br />

23<br />

24<br />

25<br />

Beim Kondensatormikrofon im Betrieb mit konstanter Ladung gilt für den<br />

Zusammenhang zwischen akustischen und elektrischen Größen<br />

± ein quadratisches Gesetz, das linearisiert wird<br />

± ein hyperbolisches Gesetz, das linearisiert wird<br />

± ein prinzipiell linearer Zusammenhang<br />

Einer mechanischen Serienschaltung aus w und m wird eine elektrische Serienschaltung<br />

aus R und C gegenübergestellt.<br />

± die analoge Schaltung ist richtig<br />

± C muß durch L ersetzt werden<br />

± es muß eine Parallelschaltung von R und C gewählt werden<br />

Welche Wellen breiten sich auch bei hohen Frequenzen mit frequenzunabhängiger<br />

Phasengeschwindigkeit aus?<br />

± Dichtewellen im unendlich ausgedehnten Körper<br />

± Biegewellen auf Stäben<br />

± Dehnwellen auf Platten<br />

Die Richtcharakteristik eines Kugelstrahlers erster Ordnung ist<br />

± eine Niere ± eine Kugel ± eine Acht<br />

Bei einer beginnenden Schwerhörigkeit werden meist<br />

± die hohen Frequenzen<br />

± die tiefen Frequenzen<br />

± alle Frequenzen gleichmäßig schlechter wahrgenommen<br />

Die Entfernungsabhängigkeit der Schallschnelle beim Kugelschallfeld nullter Ordnung ist<br />

± 1/r im Nahfeld ± 1/r im Fernfeld ± 1/r 2 im Fernfeld<br />

Eine Schwebung nennt man<br />

± den Zustand nach einer durchzechten Nacht<br />

± die Überlagerung zweier Sinusschwingungen mit geringer Frequenzdifferenz<br />

± eine stehende Welle<br />

Die Belastung eines Kugelstrahlers nullter Ordnung durch das Schallfeld ist bei tiefen<br />

Frequenzen näherungsweise<br />

± eine Masse ± eine Feder ± ein Widerstand.<br />

Unter Mithörschwelle versteht man<br />

± die Verdeckung eines Pegels durch einen anderen<br />

± einen gerade hörbaren Pegel<br />

± einen gerade hörbaren Pegel bei Anwesenheit eines anderen<br />

Ein Materieteilchen der Luft, in der sich Schall ausbreitet, bewegt sich<br />

± gar nicht ± mit ca. 340 m/s ± mit seiner Schnelle<br />

Bei einer elektroakustischen Anlage zur Sprachunterstützung in einem Hörsaal soll<br />

± man den Lautsprecher deutlich orten<br />

± der Lautsprecher möglichst laut sein<br />

± die Sprachverständlichkeit unauffällig erreicht werden


Diese Klausur gliedert sich in zwei Teile:<br />

Der Aufgabenteil enthält zwei Aufgaben, die mit insgesamt 50 Punkten bewertet werden.<br />

Der Fragenteil enthält 25 Fragen, die mit jeweils zwei Punkten bewertet werden. Von jeweils drei Antworten<br />

ist nur eine richtig. Um Manipulationen auszuschließen, wird innerhalb des Fragenteils von der hier<br />

erreichten Punktzahl für jede falsch angekreuzte Antwort ein Punkt abgezogen. Statistisches Ankreuzen<br />

ergibt damit null Punkte. Es darf nur jeweils eine Antwort angekreuzt werden.<br />

Die Klausur ist mit 50 Punkten bestanden.<br />

n<br />

FACHHOCHSCHULE KÖLN FACHBEREICH NACHRICHTENTECHNIK<br />

TECHNISCHE AKUSTIK<br />

KLAUSUR ENDE WS 2000/01 AM 27. SEPTEMBER 2000<br />

Prof. Dr.-Ing. J.L Leichsenring<br />

m<br />

v<br />

w 1<br />

w 2<br />

Aufgabe 1: (25 Punkte)<br />

Ein Lautsprecher in einem geschlossenen Gehäuse hat<br />

einen Membrandurchmesser von D = 8,5 cm und einen maximalen<br />

Membranhub x = ! 4mm (Scheitelwert).<br />

Welche akustische Wirkleistung kann der Lautsprecher bei f =<br />

90Hz höchstens abgeben?<br />

Rechnen Sie mit c = 345 m/s und ✣ = 1, 2 kg/m . 3<br />

Aufgabe 2: (25 Punkte)<br />

Gegeben ist die skizzierte mechanische Anordnung<br />

mit folgenden Bauelementen:<br />

m = 1g, n = 10 , ,<br />

−2 m<br />

w1 = 5<br />

N Ns<br />

m w2 = 1, 25 Ns<br />

m<br />

Berechnen sie die Frequenz, bei der die mechanische<br />

Impedanz reell wird.<br />

Z mech


TECHNISCHE AKUSTIK 27. SEPTEMBER 2000 Seite 4<br />

————————————————————————————————————————<br />

FRAGENTEIL<br />

1<br />

2<br />

3<br />

4<br />

5<br />

6<br />

7<br />

8<br />

9<br />

10<br />

11<br />

12<br />

Die Zeitfunktion des Schalldrucks von Vokalen ist im Prinzip<br />

± periodisch ± impulsartig ± völlig unregelmäßig<br />

Die Übertragungsfunktion eines Tieftonlautsprechers im geschlossenen Gehäuse ist die<br />

eines Hochpasses<br />

± erster ± zweiter ± dritter Ordnung<br />

Bei einer Serienschaltung aus akustischen Bauelementen ist<br />

± die Schnelle ± der Druck ± der Schallfluß<br />

in allen Schaltelementen gleich<br />

In einem Hallraum mit ideal gut reflektierenden Wänden erhält man eine endliche<br />

Nachhallzeit wegen der<br />

± endlichen Bandbreite der Anregung<br />

± endlichen Leistung der Schallquelle<br />

± Dämpfung des Schalls in Luft<br />

Trifft ein Schallereignis auf zwei Wegen mit unterschiedlicher Laufzeit bei einem Hörer<br />

ein, so ist dann ein Echo zu hören, wenn diese Laufzeit<br />

± kleiner als 10ms ist<br />

± größer als 40ms ist<br />

± zwischen 0,2ms und 2ms liegt<br />

Der Absorptionsgrad an einer Grenzfläche ist definiert als das Verhältnis der<br />

± Schalldrücke des hinlaufenden und reflektierten Schalls<br />

± Schalldrücke des absorbierten und hinlaufenden Schalls<br />

± Schallenergiedichten des absorbierten und hinlaufenden Schalls<br />

In einem mechanischen Kreis ist die Schnelle<br />

± die abgestrahlte Schallschnelle<br />

± die Geschwindigkeit, mit der sich ein mechanisches Element als Ganzes bewegt<br />

± die Differenz der Geschwindigkeiten zweier Punkte im mechanischen Kreis<br />

Wirkt auf einen zylindrischen Körper eine Kraft senkrecht auf die Stirnfläche, beschreibt<br />

der Elastizitätsmodul E den Zusammenhang zwischen<br />

± Dehnung und Querkontraktion<br />

± Kraft und Normalspannung<br />

± Normalspannung und Dehnung<br />

Die Schallgeschwindigkeit von Biegewellen in Stäben mit quadratischem Querschnitt<br />

± steigt ± fällt ± ändert sich nicht<br />

bei steigender Frequenz<br />

Wenn man Sprache in einem PCM-System überträgt und diese mit der A-Kompandierungskurve<br />

nach CCITT komprimiert, so handelt es sich um eine Übertragung mit<br />

± parametrischer Codierung<br />

± Signalformcodierung<br />

± Textanalyse und -Sythese<br />

In einer stehenden Welle verhalten sich die Schalldrücke in den Minima und Maxima wie<br />

3:5, der Reflexionsfaktor ist dann<br />

± 0,25 ± 0,6 ± 1,5<br />

Den Schalldruck kann man beschreiben durch einen<br />

± räumlichen Vektor ± Skalar ± konstanten Wert


TECHNISCHE AKUSTIK 27. SEPTEMBER 2000 Seite 5<br />

————————————————————————————————————————<br />

FRAGENTEIL<br />

13<br />

14<br />

15<br />

16<br />

17<br />

18<br />

19<br />

20<br />

21<br />

22<br />

23<br />

24<br />

25<br />

Unter Schallfluß versteht man das Produkt aus<br />

± Schnelle v und Querschnittsfläche S in akustischen Filtern<br />

± Schnelle v und Druck p im Schallfeld<br />

± Intensität I und Schallgeschwindigkeit c im Schallfeld<br />

Bei stimmhaften Lauten der Sprache erwachsener Menschen liegt die Grundfrequenz in<br />

der Gegend von<br />

± 50 Hz ± 150 Hz ± 1000 HZ<br />

Ein Tauchspulmikrofon ist ein<br />

± elektromagnetischer Wandler<br />

± Wasserschallwandler<br />

± elektrodynamischer Wandler<br />

Die mech. Impedanz ist<br />

± Schnelle/Kraft ± Kraft/Schnelle ± Schalldruck/Schallfluß<br />

Zwischen zwei schallharten Wänden findet man die niedrigste Eigenfrequenz, wenn der<br />

Wandabstand<br />

± ½ ± ¼ ± 1/1 der Schallwellenlänge ist<br />

Bei der Richtcharakteristik eines Rohrschlitzmikrofons ergeben sich „Nebenzipfel”<br />

± bei allen Frequenzen<br />

± nur unterhalb einer bestimmten Frequenz<br />

± nur oberhalb einer bestimmten Frequenz<br />

Der Pegel, bei dem ein Ton bei Verdeckung durch ein Geräusch gerade hörbar wird, ist<br />

die<br />

± Mithörschwelle ± Hörschwelle ± Fühlschwelle<br />

Eine Kugelwelle erster Ordnung ist<br />

± eine besonders starke Kugelwelle<br />

± eine von einem Dipolstrahler erzeugte Welle<br />

± eine Kugelwelle bei kr = 1<br />

Die Sabinesche Nachhallformel gilt als Näherung in Räumen<br />

± mit geringer Schallabsorption<br />

± mit hoher Schallabsorption<br />

± mit großen Abmessungen<br />

Im Nahfeld eines Kugeslstrahlers nullter Ordnung<br />

± überwiegt die akustische Blindleistung<br />

± überwiegt die akustische Wirkleistung<br />

± verschwindet die akustische Leistung<br />

Unter „Hörschwelle” versteht man den Schallpegel, bei dem Schall gerade<br />

± wahrgenommen wird ± unangenehm laut wird ± verdeckt wird<br />

Der Schalldruck bei Luftschall ist die Druckänderung aufgrund von<br />

± wetterbedingten Druckschwankungen<br />

± Schallschwingungen<br />

± Schallpegeländerungen<br />

Ein Mikrofon, das als Schnelleempfänger arbeitet, hat die Richtcharakteristik einer<br />

± Kugel ± Niere ± Acht


Diese Klausur gliedert sich in zwei Teile:<br />

Der Aufgabenteil enthält drei Aufgaben, die mit insgesamt 50 Punkten bewertet werden.<br />

Der Fragenteil enthält 25 Fragen, die mit jeweils zwei Punkten bewertet werden. Von jeweils drei Antworten<br />

ist nur eine richtig. Um Manipulationen auszuschließen, wird innerhalb des Fragenteils von der hier<br />

erreichten Punktzahl für jede falsch angekreuzte Antwort ein Punkt abgezogen. Statistisches Ankreuzen<br />

ergibt damit null Punkte. Es darf nur jeweils eine Antwort angekreuzt werden.<br />

Die Klausur ist mit 50 Punkten bestanden.<br />

1m<br />

1,5m<br />

gFACHHOCHSCHULE KÖLN FACHBEREICH NACHRICHTENTECHNIK<br />

TECHNISCHE AKUSTIK<br />

KLAUSUR ENDE SS 2000 AM 07. JULI 2000<br />

Prof. Dr.-Ing. J.L Leichsenring<br />

1cm<br />

50µm<br />

2m<br />

Aufgabe 1: (20 Punkte)<br />

Vor einer Wand mit einem frequenzunabhängigen Reflexionsfaktor<br />

R = 0,98 befindet sich eine Schallquelle. Sie gibt ein bandbegrenztes<br />

Rauschen im Bereich 0, 5 kHz [ f [ 5kHzmit<br />

einer akustischen<br />

Leistung von Pak = 40 µW ab.<br />

Bestimmen Sie die Schallintensität J, den Schallpegel L und den<br />

Schalldruck p am Meßort.<br />

Aufgabe 2: (15 Punkte)<br />

Gegeben ist ein Stab aus Aluminium ( E = 7 $ 10 ,<br />

10 N/m2 ✯ = 2700 kg/m ) mit quadratischem Querschnitt bei einer Kanten-<br />

3<br />

länge h = 1 cm. Ein Ende des Stabes ist reflexionsfrei abgeschlossen,<br />

das andere wird mit einer Biegewelle erregt, die die<br />

Frequenzen 1 kHz und 2 kHz enthält.<br />

a) Welcher Frequenzanteil durchläuft den Stab schneller?<br />

b) Wie groß ist die Laufzeitdifferenz zwischen beiden Frequenzanteilen,<br />

nachdem eine Strecke von 1m durchlaufen wurde?<br />

Aufgabe 3: (15 Punkte)<br />

Gegeben sind die in der Skizze eingetragenen Abmessungen eines<br />

Kondensatormikrofons. Weiter ist<br />

ρ0 = 1,2 kg/m 3 , c = 345 m/s, ε0 = 8,86 pF/m<br />

Welche Polarisierungsspannung U0 muß man anlegen, um einen<br />

Leerlauf-Druckübertragungsfaktor Tp = 10 -3 V/Pa zu erhalten?


TECHNISCHE AKUSTIK 07. JULI 2000 Seite 5<br />

————————————————————————————————————————<br />

FRAGENTEIL<br />

1<br />

2<br />

3<br />

4<br />

5<br />

6<br />

7<br />

8<br />

9<br />

10<br />

11<br />

12<br />

13<br />

Die Stelle im Vokaltrakt, wo ein Laut entsteht, bezeichnet man als<br />

± Artikulationsort ± Stimmritze ± Formant<br />

Der Wandlerfaktor N eines idealen N-Wandlers verknüpft<br />

± F mit I und v mit U<br />

± F mit U und v mit I<br />

± alle elektrischen und mechanischen Größen<br />

Zwei Rohre mit unterschiedlichem Durchmesser sind miteinander verbunden. An dem<br />

Querschittssprung findet man den Reflexionsfaktor Null, wenn hier die<br />

± mechanischen ± spezifischen ± akustischen Impedanzen gleich sind.<br />

Um bei einem rauschähnlichen Signal mehr als 99% des Zeitverlaufs unverfälscht zu<br />

übertragen, muß der Kanal Spitzenwerte vom<br />

± 10 fachen ± 3 fachen ± 1 fachen<br />

Effektivwert unverzerrt übertragen können.<br />

Bei Intensitässtereophonie sei bei sonst gleichem Signal der rechte Lautsprecher 5 dB<br />

lauter. Die Phantomquelle liegt dann etwa<br />

± 10 ° nach links ± in der Mitte ± 10 ° nach rechts<br />

Eine Schallwelle wird vollständig reflektiert, wenn die Schallkennimpedanz Z0 des<br />

Mediums, auf das der Schall trifft<br />

± angepaßt ist ± schallweich ist ± reell ist<br />

Bei einer Feder ist die Kraft proportional<br />

± zur Beschleunigung ± zur Schnelle ± zur Auslenkung<br />

Eine konstante Phasengeschwindigkeit findet man bei<br />

± Torsionswellen<br />

± Biegewellen auf Platten<br />

± Biegewellen auf Stäben<br />

Unter Schnelle versteht man die<br />

± Ausbreitungsgeschwindigkeit des Schalls<br />

± Windgeschwindigkeit<br />

± Momentangeschwindigkeit eines Materieteilchens<br />

Will man Sprache ohne Dynamikkompression übertragen, muß man eine Dynamik von<br />

etwa<br />

± 20 dB ± 40 dB ± 65 dB vorsehen<br />

In gasförmigen Medien können auftreten<br />

± Schubspannungen ± Druckspannungen ± Torsionen<br />

Unter "Schallkennimpedanz" versteht man<br />

± die spez. Schallfeldimpedanz in einer ebenen Welle<br />

± die spez. Schallfeldimpedanz auf der Oberfläche eines Schallstrahlers<br />

± ein Merkmal, an dem man einen Sprecher identifizieren kann<br />

Ein kurzer Exponentialtrichter überträgt unterhalb seiner Grenzfrequenz<br />

± keine ± sehr wenig ± maximale<br />

Wirkleistung.


TECHNISCHE AKUSTIK 07. JULI 2000 Seite 6<br />

————————————————————————————————————————<br />

FRAGENTEIL<br />

14<br />

15<br />

16<br />

17<br />

18<br />

19<br />

20<br />

21<br />

22<br />

23<br />

24<br />

25<br />

Schallabsorption bei der Schallausbreitung in Luft beeinflußt die Nachhallzeit eines<br />

Raums<br />

± bei hohen Frequenzen ± bei tiefen Frequenzen ± gar nicht<br />

Die untere Grenzfrequenz eines dynamischen Tieftonlautsprechers liegt<br />

± bei kR = 1<br />

± bei der mech. Resonanzfrequenz<br />

± bei der ersten Eigenfrequenz des Gehäuses<br />

Die Parallelschaltung aus zwei gleichen mechanischen Federn hat die<br />

± doppelte<br />

± gleiche<br />

± halbe Nachgiebigkeit einer einzelnen Feder<br />

Der Schubmodul beschreibt das Verhalten von<br />

± Tänzern in einer engen Diskothek<br />

± PKWs in einem Autobahnstau<br />

± Materialien unter Scherbeanspruchung<br />

Die Richtcharakteristik eines Kugelstrahlers 1. Ordnung ist<br />

± eine Kugel ± eine Niere ± eine Acht<br />

Die Gesamtlautstärke zweier Geräusche, die in unterschiedlichen Frequenzgruppen<br />

liegen, kann man berechnen aus der Summe der<br />

± Lautheiten ± Lautstärken ± Intensitäten<br />

In großer Entfernung von einem Kugelstrahler 1. Ordnung sinkt der Schalldruck<br />

proportional zu<br />

± 1/r ± 1/r 2 ± 1/r 3<br />

Die Grenzfrequenzen verschiedener Oktavfilter<br />

± verhalten sich wie fo/fu = 2<br />

± verhalten sich wie fo/fu = 3 2<br />

± haben einen konstanten Abstand fo − fu<br />

Interferenzen im Nahfeld eines Kolbenstrahlers treten auf<br />

± bei tiefen Frequenzen<br />

± bei hohen Frequenzen<br />

± bei hohen Amplituden<br />

Die Einheit für den Lautstärkepegel ist<br />

± dB ± phon ± sone<br />

In einer stehenden Welle kann man die Welligkeit m messen. Sie ist das Verhältnis der<br />

± Schallkennimpedanzen<br />

± Reflexionsfaktoren<br />

± eff. Schalldrücke<br />

in den Minima und Maxima der stehenden Welle<br />

Um eine Mikrofonkalibrierung nach dem Reziprozitätsverfahren durchführen zu können,<br />

muß einer der verwendeten elektroakustischen Wandler<br />

± als Sender und Empfänger zu betreiben sein<br />

± geeicht sein<br />

± ein N-Wandler sein


Diese Klausur gliedert sich in zwei Teile:<br />

Der Aufgabenteil enthält drei Aufgaben, die mit insgesamt 50 Punkten bewertet werden.<br />

Der Fragenteil enthält 25 Fragen, die mit jeweils zwei Punkten bewertet werden. Von jeweils drei Antworten<br />

ist nur eine richtig. Um Manipulationen auszuschließen, wird innerhalb des Fragenteils von der hier<br />

erreichten Punktzahl für jede falsch angekreuzte Antwort ein Punkt abgezogen. Statistisches Ankreuzen<br />

ergibt damit null Punkte. Es darf nur jeweils eine Antwort angekreuzt werden.<br />

Die Klausur ist mit 50 Punkten bestanden.<br />

F<br />

b<br />

FACHHOCHSCHULE KÖLN FACHBEREICH NACHRICHTENTECHNIK<br />

TECHNISCHE AKUSTIK<br />

KLAUSUR ENDE WS 1999/2000 AM 7. FEBRUAR 2000<br />

Prof. Dr.-Ing. J.L. Leichsenring A<br />

Aufgabe 1: (10 Punkte)<br />

Für ein dynamisches Mikrofon ist gegeben:<br />

Membranfläche: S = 5 cm 2 Membranmasse: mM = 0,2 g<br />

Membranresonanz: f0 = 500 Hz Kreisgüte: Q = 0,2<br />

magnet. Induktion: B = 1 T<br />

Bestimmen Sie die Länge der Drahtwicklung , wenn das Mikrofon einen Leerlauf-Druckübertragungsfaktor<br />

T = 1 mV/Pa haben soll.<br />

w1<br />

n<br />

m<br />

l<br />

w 2<br />

h<br />

Aufgabe 2: (20 Punkte)<br />

In der mechanischen Schaltung ist m = 1g , n = 10 -3 m/N und<br />

w1 = w2 = 1 Ns/m.<br />

a) Geben Sie die Gleichung für die mech. Impedanz Z an.<br />

b) Bestimmen Sie die Kennfrequenz f0 (ohne Dämpfung!)<br />

der Schaltung und die komplexe Impedanz bei dieser<br />

Frequenz.<br />

c) Geben sie die elektrische Ersatzschaltung mit den Werten<br />

aller Bauelemente an, wenn der Zusammenhang F = M·I<br />

und U = M·v mit M = 1N/A gilt.<br />

Aufgabe 3: (20 Punkte)<br />

In einem Raum mit den Abmessungen l = 7,3m, b = 5m, h<br />

= 2,7m wird eine Nachhallzeit T = 2,1s gemessen. In dem<br />

Raum befindet sich ein Lüfter, der einen Schallpegel L1<br />

= 71 dB erzeugt.<br />

Sie haben die Aufgabe, den Schallpegel auf L2 = 65 dB zu<br />

senken, indem sie die Schallabsorption des Raums erhöhen.<br />

Sie planen einen Teppichboden mit α = 0,07, drei Seitenwände<br />

mit α = 0,11 und eine gut absorbierende Deckenverkleidung.<br />

Die große Seitenwand bleibt im gegebenen<br />

Zustand.<br />

Welchen Absorptionsgrad muß die Deckenverkleidung<br />

haben, um die Pegelminderung zu erreichen?


Fragenteil: 07. FEB 2000<br />

0l)<br />

02)<br />

03)<br />

04)<br />

05)<br />

06)<br />

07)<br />

08)<br />

09)<br />

10)<br />

11)<br />

12)<br />

Stellt man den zeitlichen Verlauf eines Sprachspektrums als Graustufen in<br />

Abhängigkeit von Zeit und Frequenz dar, so erhält man<br />

± ein Spektrogramm ± eine Formantkurve ± ein Cepstrum<br />

Beim Kondensatormikrofon im Betrieb mit konstanter Ladung gilt für den<br />

Zusammenhang zwischen akustischen und elektrischen Größen<br />

± ein prinzipiell linearer Zusammenhang<br />

± ein quadratisches Gesetz, das linearisiert wird<br />

± ein hyperbolisches Gesetz, das linearisiert wird<br />

Die akustische Impedanz ist definiert als<br />

± Schalldruck p / Schallschnelle v<br />

± Kraft F / Schallfluß q<br />

± Schalldruck p / Schallfluß q<br />

Die Grenzfrequenzen verschiedener Oktavfilter<br />

± haben einen konstanten Abstand f0 - fu<br />

± verhalten sich wie fo / fu = 2<br />

± verhalten sich wie fo / fu = 3 2<br />

Für einem N-Wandler kennen Sie die Größen des elektrischen und<br />

mechanischen Ersatzbildes. Den Wandlerfaktor N erhalten Sie als<br />

± N = w $ R ± N = m/L ± N= m/L<br />

Luf:schallschnelle ist die Geschwindigkeit, mit der sich<br />

± Luftschall ausbreitet<br />

± die Schallquelle bewegt<br />

± ein Materieteilchen bewegt<br />

Bei einem mech. Widerstand ist die Schnelle proportional<br />

± zur Kraft<br />

± zur Beschleunigung<br />

± zum Ausschlag<br />

Der Elastizitätsmodul eines Materials gibt den Zusammenhang zwischen<br />

± Kraft und Schubspannung<br />

± Dehnung und Querkontraktion<br />

± Kraft und Dehnung an.<br />

Bei zwei in Serie geschalteten Massen m1 = m2 = 10g findet man<br />

± die Gesamtmasse 20g<br />

± die Gesamtmasse 5g<br />

± daß die Serienschaltung nicht möglich ist<br />

Morphene sind<br />

± Schmerzmittel<br />

± Bausteine der Sprache<br />

± spezielle Kuststoffe zur Membranherstellung<br />

Ein beidseitig offenes Rohr wird durch eine obertonreiche Pulsfolge mit der Periode<br />

der tiefsten Eigenfrequenz des Rohrs angeregt. Es werden<br />

± alle ± nur ungeradzahlige ± nur geradzahlige<br />

Obertöne als Eigenfrequenzen des Rohrs angeregt<br />

Eine Schallwelle in einem akustischen Meßrohr trifft auf einen schallweichen<br />

Abschluß. Hier wird<br />

± die gesamte Schallenergie absorbiert<br />

± die gesamte Schallenergie reflektiert<br />

± genau die Hälfte der Schallenergie reflektiert


Fragenteil: 07. FEB 2000<br />

13)<br />

14)<br />

15)<br />

16)<br />

17)<br />

18)<br />

19)<br />

20)<br />

21)<br />

22)<br />

23)<br />

24)<br />

25)<br />

Die Kammern eines Schalldämpfers für Verbrennungsmotoren bilden einen akustischen<br />

± Hochpaß ± Tiefpaß ± Bandpaß<br />

Die äquivalente Absorptionsfläche eines Raums ist<br />

± die Schalldämmung einer Wand<br />

± das gesamte Absorptionsvermögen des Raums<br />

± die Schallabsorption durch die Luft<br />

Ein Druckgradienmikrofon ist ein Mikrofon mit<br />

± Achtercharakteristik ± Nierencharakteristik ± Kugelcharaktenstik<br />

Das Produkt Kraft @ Schnelle beschreibt eine<br />

± mechanische Impedanz<br />

± mechanische Leistung<br />

± spezifische Schallfeldimpedanz<br />

Die Schallgeschwindigkeit von Biegewellen in runden Stäben<br />

± ist proportional zur Stablänge<br />

± wächst mit steigendem Stabdurchmesser<br />

± hängt nicht von den Abmessungen ab<br />

Eine Nierencharakteristik erhält man durch Überlagerung der Schallfelder<br />

± zweier Kugelstrahler nullter Ordnung<br />

± zweier Kugelstrahler erster Ordnung<br />

± je eines Kugelstrahlers erster und nullter Ordnung<br />

Unter Hörschwelle versteht man den Schallpegel, bei dem Schall gerade<br />

± wahrgenommen wird ± unangenehm laut wird ± verdeckt wird<br />

Kolbenstrahler kann man zur Berechnung<br />

± in jedem Fall durch Kugelstrahler ersetzen<br />

± nur durch Kugelstrahler nullter Ordnung ersetzen<br />

± für tiefe Frequenzen durch einen entsprechenden Kugelstrahler nähern<br />

Auch für den theoretischen Fall ideal schallharter Wände ist die Höhe der Nachhallzeit<br />

eines Raums bei hohen Frequenzen begrenzt durch<br />

± das menschliche Hörvermögen<br />

± die Richtwirkung des Meßmikrofons<br />

± die Ausbreitungsverlus^in Luft<br />

Die Flächen gleicher Phase sind. bei Kugelwellen nullter Ordnung<br />

± Ebenen ± Kugelflächen ± Zylinderflächen<br />

Ein der Empfindung der Lautstärke proportionales Maß ist<br />

± die Lautheit in sone<br />

± der Lautstärkepegel in phon<br />

± der Schalldruckpegel in dB<br />

Schall kann sich nicht ausbreiten<br />

± im luftleeren Raum ± in Luft ± in festen Körpern<br />

Der Dynamikbereich eines Mikrofons ist zu niederigen Schallpegeln hin begrenzt durch<br />

± den Übertragungsfaktor des Mikrofons<br />

± Störspannungen<br />

± die Masse der Membran


FACHHOCHSCHULE KÖLN FACHBEREICH NACHRICHTENTECHNIK<br />

TECHNISCHE AKUSTIK<br />

KLAUSUR ANFANG WS 1999/2000 AM 28. SEPTEMBER 1999<br />

Prof. Dr.-Ing. J.L Leichsenring A<br />

Diese Klausur gliedert sich in zwei Teile:<br />

Der Aufgabenteil enthält zwei Aufgaben, die mit insgesamt 50 Punkten bewertet werden.<br />

Der Fragenteil enthält 25 Fragen, die mit jeweils zwei Punkten bewertet werden. Von jeweils drei Antworten<br />

ist nur eine richtig. Um Manipulationen auszuschließen, wird innerhalb des Fragenteils von der hier erreichten<br />

Punktzahl für jede falsch angekreuzte Antwort ein Punkt abgezogen. Statistisches Ankreuzen ergibt<br />

damit null Punkte. Es darf nur jeweils eine Antwort angekreuzt werden.<br />

Die Klausur ist mit 50 Punkten bestanden.<br />

durchtretende<br />

Welle<br />

Leichtbauwand<br />

einfallende<br />

Welle<br />

reflektierte<br />

Welle<br />

Aufgabe 1: (25 Punkte)<br />

Gegeben ist ein Tieftonlautsprecher im geschlossenen Gehäuse, das<br />

im Übertragungsbreich klein gegen die Wellenlänge ist.<br />

Membrandurchmesser D = 8 cm ρ 0 = 1,2 kg/m 3<br />

Wicklungswiderstand R 1 = 4 Ω c = 345 m/s<br />

Membranmasse m M = 5 g<br />

Wandlerfaktor M = 5 N/A<br />

Bestimmen Sie den elektroakustischen Wirkungsgrad im Übertragungsbereich.<br />

(Vernachlässigen sie dabei die Belastung der<br />

Membranrückseite)<br />

Aufgabe 2: (25 Punkte)<br />

Gegeben ist eine Leichbauwand mit einem Flächengewicht<br />

m* = 60 kg/m 3 .<br />

Bestimmen sie den Reflexionsfaktor R und den Schallabsorptionsgrad<br />

a für eine senkrecht auf die Wand treffende ebene Schallwelle<br />

bei f = 150 Hz und daraus die Pegelabnahme des Schalls bei<br />

durchtritt durch die Wand.<br />

Hinweis: Die Abschlußimpedanz für die einfallende Welle ist die<br />

Summe aus der Impedanz der Wand und der Kennimpedanz der<br />

Luft dahinter. Berechnen Sie R auf midestens 6 Stellen genau.


Name: ______________________________________________ Matrikelnummer: ___________ - 2 -<br />

————————————————————————————————————————<br />

Fragenteil: WS 1999/2000 A A<br />

01)<br />

02)<br />

03)<br />

04)<br />

05)<br />

06)<br />

07)<br />

08)<br />

09)<br />

10)<br />

11)<br />

12)<br />

13)<br />

Zwei parallelgeschaltete Federn mit den Nachgiebigkeiten n1 = 6 m/N und n2 = 3 m/N haben die<br />

Gesamtnachgiebigkeit<br />

± 9,00 m/N ± 2,00 m /N ± 0,50 m/N<br />

Die Schallenergiedichte ist definiert als<br />

± Schalleistung je Flächeneinheit<br />

± Schallschnelle mal Fläche<br />

± Schallenergie je Volumeneinheit<br />

Ein Biegeschwinger ist<br />

± ein Federpendel<br />

± eine Form piezoelektrischer Wandler<br />

± die Membran eines Kondensatormikrofons<br />

Ein dynamischer Tieftonlautsprecher hat eine Resonanzfrequenz f0 = 45Hz. Beim Einbau in ein geschlossenes<br />

Gehäuse wird seine Resonanzfrequenz<br />

± gleichbleiben ± sinken ± steigen<br />

Die Phasengeschwindigkeit von Biegewellen in Platten ist<br />

± unabhängig von f ± proportional zu f ± proportional zu pf<br />

Der Bereich der Pegelunterschiede zwischen den beiden Lautsprechersignalen bei Intensitätsstereophonie<br />

liegt bei etwa<br />

± "5 dB ± "20 dB ± "60 dB<br />

In einem Kugelschallfeld nullter Ordnung<br />

± überwiegt im Nahfeld die Blindleistung<br />

± treten im Nahfeld Interferenzen auf<br />

± ist es im Nahfeld besonders ruhig<br />

Die Schallgeschwindigkeit von Torsionswellen in Stäben mit quadratischem Querschnitt<br />

± steigt ± fällt ± ändert sich nicht bei steigender<br />

Frequenz<br />

Einer Verdopplung des Lautstärkeeindrucks entspricht eine Erhöhung der Lautheit von 4 sone nach<br />

± 5 sone ± 15 sone ± 8 sone<br />

Akustische Schaltelemente werden nur verwendet, wenn<br />

± die Abmessungen klein gegen die Wellenlänge sind<br />

± die Schalldruckamplituden klein bleiben<br />

± keine Verluste auftreten<br />

Wir hören nichtlineare Verzerrungen, die im Ohr selbst entstehen,<br />

± weder bei hohen noch bei niedrigen Lautstärken<br />

± bei sehr niedrigen Lautstärken<br />

± bei hohen Lautstärken<br />

Eine mechanische Impedanz der Größe Zmech = 4 beschreibt eine<br />

± starre ± fehlende ± elastische<br />

Verbindung zwischen zwei "Klemmen" einer mechanischen Struktur<br />

Der Absorptionsgrad an einer Grenzfläche ist definiert als Verhältnis der Schallenergiedichten des<br />

± hinlaufenden und reflektierten<br />

± reflektierten und hinlaufenden<br />

± absorbierten und hinlaufenden Schalls.


Name: ______________________________________________ Matrikelnummer: ___________ - 3 -<br />

————————————————————————————————————————<br />

Fragenteil: WS 1999/2000 A A<br />

14)<br />

15)<br />

16)<br />

17)<br />

18)<br />

19)<br />

20)<br />

21)<br />

22)<br />

23)<br />

24)<br />

25)<br />

Lautheit ist eine<br />

± Bezeichnung für besonders hohe Lautstärken<br />

± andere Bezeichnung für die Lautstärke<br />

± dem Lautstärkeeindruck proportionale Größe<br />

Die menschliche Stimme erzeugt Schalleistungen in der Größenordnung von<br />

± 1 W ± 1 mW ± 1 µW<br />

Die Entfernungsabhängigkeit der Wirkintensität im Nahfeld eines Kugelstrahlers nullter Ordnung ist<br />

proportional zu<br />

± r ± 1/r ± 1/r 2<br />

Die Mündungskorrektur für ein offenes Rohrende berücksichtigt, daß sich das Rohr bez. seiner Eigentöne<br />

verhält, als sei es<br />

± viel länger ± etwas länger ± etwas kürzer<br />

als seine tatsächliche Länge beträgt<br />

Bei der Wiedergabe von Schall über mehrere Lautsprecher werden Echos hörbar, wenn die Laufzeit des<br />

Schalls zwischen zwei Lautsprechern<br />

± weniger als 50 ms ± mehr als 50 ms ± mehr als 1 ms beträgt.<br />

Die Länge eines Exponetialtrichters hat Einfluß auf ± seine untere Grenzfrequenz<br />

± die Schwankungen im Übertragungsbereich<br />

± seine obere Grenzfrequenz<br />

Die Richtcharakteristik eines Kugelstrahlers erster Ordnung ist<br />

± eine Acht ± eine Kugel ± eine Niere<br />

An einem schallweichen Abschluß<br />

± tritt keine Reflexion auf ± ist | R | = 1 ± ist die Absorption optimal<br />

Die Sabinesche Nachhallformel gilt als Näherung in Räumen<br />

± mit geringer Schallabsorption ± mit hoher Schallabsorption ± mit großen Abmessungen<br />

Die Zahl der bei einem Hörer in einem Raum eintreffenden Schallrückwürfe steigt an mit der Zeit<br />

± linear ± mit der dritten Potenz ± quadratisch<br />

Bei der Richtcharakteristik eines Rohrschlitzmikrofons ergeben sich "Nebenzipfel"<br />

± bei allen Frequenzen<br />

± nur unterhalb einer bestimmten Frequenz<br />

± nur oberhalb einer bestimmten Frequenz<br />

Normalbeanspruchung ist die Beanspruchung eines festen Körpers, die<br />

± senkrecht zur Bezugsfläche<br />

± normalerweise<br />

± parallel zur Bezugsfläche auftritt


FACHHOCHSCHULE KÖLN FACHBEREICH NACHRICHTENTECHNIK<br />

TECHNISCHE AKUSTIK<br />

KLAUSUR ENDE SS 1999 AM 24. JUNI 1999<br />

Prof. Dr.-Ing. J.L Leichsenring A<br />

Diese Klausur gliedert sich in zwei Teile:<br />

Der Aufgabenteil enthält drei Aufgaben, die mit insgesamt 50 Punkten bewertet werden.<br />

Der Fragenteil enthält 25 Fragen, die mit jeweils zwei Punkten bewertet werden. Von jeweils drei Antworten<br />

ist nur eine richtig. Um Manipulationen auszuschließen, wird innerhalb des Fragenteils von der hier erreichten<br />

Punktzahl für jede falsch angekreuzte Antwort ein Punkt abgezogen. Statistisches Ankreuzen ergibt<br />

damit null Punkte. Es darf nur jeweils eine Antwort angekreuzt werden.<br />

Die Klausur ist mit 50 Punkten bestanden.<br />

w 1<br />

m<br />

w 2<br />

Aufgabe 2: (15 Punke)<br />

n<br />

Aufgabe 1: (20 Punkte)<br />

Für eine mechanische Schaltung sind gegeben:<br />

m = 1 g, n = 10 -3 m/N, w 1 = 20 Ns/m und w 2 = 5 Ns/m<br />

a) Fassen Sie, wo möglich, gleichartige Elemente zusammen,<br />

zeichnen sie die vereinfachte Schaltung und geben<br />

sie die Funktion der mechanischen Impedanz Z mech an.<br />

b) Berechnen Sie die Frequenz, bei der Z mech reell wird.<br />

c) Geben Sie den Wert von Z mech bei der unter b) berechneten<br />

Frequenz an.<br />

In der Frequenzgruppe um f = 500 Hz wird eine Lautheit s 500 = 11,3 sone und in der Frequenzgruppe<br />

um f = 2000 Hz s 4000 = 4,7 sone gemessen.<br />

Bestimmen Sie die gesamte Lautheit sges und den gesamten Lautstärkepegel Lsges .<br />

Aufgabe 3: (15 Punke)<br />

In einem Wohnraum mit den Abmessungen 6m · 4,5m · 2,7m mißt man eine Nachhallzeit<br />

T = 0,7s.<br />

Welche mittlere elektrische Leistung müssen Sie einem Lautsprecher mit einem elektroakustischen<br />

Wirkungsgrad η = 0,7% zuführen, um im diffusen Schallfeld einen Schallpegel L = 90 dB zu erzeugen<br />

?


Name: ______________________________________________ Matrikelnummer: ___________ - 4 -<br />

————————————————————————————————————————<br />

Fragenteil: SS 1999 E A<br />

1)<br />

2)<br />

3)<br />

4)<br />

5)<br />

6)<br />

7)<br />

8)<br />

9)<br />

10)<br />

11)<br />

12)<br />

13)<br />

In einem mechanischen Kreis ist die Schnelle<br />

± die abgestrahlte Schallschnelle<br />

± die Geschwindigkeit, mit der sich ein mechanisches Element als Ganzes bewegt<br />

± die Differenz der Geschwindigkeiten zweier Punkte im mechanischen Kreis<br />

Der Reflexionsfaktor 0 tritt auf, wenn Schall auf eine Grenzfläche mit<br />

± angepaßtem ± schallweichem ± komplexem Abschluß trifft<br />

In einem akustischen Meßrohr, das mit einem Reflexionsfaktor R = 0,8 abgeschlossen ist, verhalten sich<br />

die Maxima der stehenden Welle zu den Minima wie<br />

± 0,8:1 ± 5:1 ± 9:1<br />

Will man Sprache ohne Dynamikkompression übertragen, muß man eine Dynamik von etwa<br />

± 20 dB ± 40 dB ± 65 dB vorsehen<br />

In Kugelwellen nullter Ordnung hat die Schallschnelle<br />

± die Richtung des Kugelradius<br />

± die Richtung der Tangente an die Kugel<br />

± eine Radial- und Tangentialkomponente<br />

Zur Bestimmung der Phasengeschwindigkeit von Dichtewellen in einem unendlich ausgedehnten Körper<br />

benötigt man den<br />

± Elastizitätsmodul E ± Kompressionsmodul K ± Longitudinalwellenmodul L<br />

Die Einheit N s m -5 ist die Einheit der<br />

± akustischen Impedanz ± Dichte ± Energiedichte<br />

Unser Gehör zeigt deutliche nichtlineare Verzerrungen<br />

± bei sehr geringen Pegeln<br />

± bei hohen Pegeln<br />

± weder bei hohen noch bei geringen Pegeln<br />

Eine Schallwelle wird vollständig reflektiert, wenn die Schallkennimpedanz Z0 des Mediums, auf das<br />

der Schall trifft<br />

± angepaßt ist ± schallweich ist ± reell ist<br />

Man vergleiche die Richtcharakteristiken eines elektroakustischen Wandlers beim Betrieb als Sender<br />

und als Empfänger<br />

± die Richtcharakteristik ist ein beiden Fällen gleich<br />

± der Sender zeigt eine schärfere Bündelung<br />

± der Empfänger zeigt eine schärfere Bündelung<br />

Einer Oktave entspricht eine<br />

± Verdopplung der Lautheit ± Verdopplung der Frequenz ± Verdopplung der Tonheit<br />

Für einen N-Wandler gilt die Beziehung<br />

± Zel = N 2 ·Zmech ± Zel = Zmech/ N 2 ± Zel = N 2 /Zmech<br />

Die Bezugrgröße für den Pegel der Schallschnelle ist<br />

± 10 -12 m/s ± 2·10 -5 m/s ± 5·10 -8 m/s


Name: ______________________________________________ Matrikelnummer: ___________ - 5 -<br />

————————————————————————————————————————<br />

Fragenteil: SS 1999 E A<br />

14)<br />

15)<br />

16)<br />

17)<br />

18)<br />

19)<br />

20)<br />

21)<br />

22)<br />

23)<br />

24)<br />

25)<br />

Ein reiner Ton ist ein Schallereignis<br />

± das von einem Musikinstrument erzeugt wird<br />

± mit sinusförmigem Schwingungsverlauf<br />

± mit einem komplizierten Spektrum<br />

In einer mechanischen "Schaltung" gilt die Knotenregel für<br />

± die Schnellen ± die Kräfte ± die Ausschläge<br />

Das akustische Ersatzbild eines Helmhotzresonators ist<br />

± ein Parallelkreis<br />

± ein Serienkreis<br />

± eine Parallelschaltung aus Masse und Widerstand<br />

Der Wirkungsgrad eines dynamischen Lautsprechers im geschlossenen Gehäuse liegt in der Größenordnung<br />

± 75% ± 20% ± 1%<br />

Eine stabförmige Materialprobe mit Kreisquerschnitt wird einer Normalbeanspruchung ausgesetzt.<br />

Positive Querdehnung tritt auf, wenn dabei die Probe<br />

± kürzer ± länger ± verbogen wird<br />

An einer Feder sind Kraft und Schnelle<br />

± in Phase ± 90 o in der Phase verschoben ± es kann keine Schnelle auftreten<br />

Ein Elektretmikrofon ist ein<br />

± elektrostatischer ± piezoelektrischer ± elektrodynamischer Wandler<br />

Der Verlauf des normierten Übertragungsmaßes ∆ L über f / fT hängt bei einem Tieftonlautsprecher im<br />

geschlossenen Gehäuse<br />

± von der Gehäusegröße ± vom Membrandurchmesser ± von der Kreisgüte ab<br />

Bei einer beginnenden Schwerhörigkeit werden meist<br />

± die hohen Frequenzen<br />

± die tiefen Frequenzen<br />

± alle Frequenzen gleichmäßig schlechter wahrgenommen<br />

Will man die Sprache eines Redners in einem Raum verstärken und verwendet dazu bei Lautsprecher und<br />

Mikrofon Kugelcharakteristiken, so erhält man ohne zusätzliche Maßnahmen Pegelanhebungen bis zu<br />

± 1,5 dB ± 10 dB ± 20 dB<br />

Im Schallfeld eines Kugelstrahlers nullter Ordnung ist bei kr >>1 der Realteil der Schallschnelle<br />

± groß gegen den Imaginärteil<br />

± klein gegen den Imaginärteil<br />

± gleich groß wie der Imaginärteil<br />

Die Eigentondichte (Eigentöne je Hz) in einem Raum steigt bei hohen Frequenzen proportional<br />

± zu f ± zu f 2 ± zu f 3


FACHHOCHSCHULE KÖLN FACHBEREICH NACHRICHTENTECHNIK<br />

TECHNISCHE AKUSTIK<br />

KLAUSUR ENDE WS 1998/99 AM 8. FEBRUAR 1999<br />

Prof. Dr.-Ing. J.L Leichsenring A<br />

Diese Klausur gliedert sich in zwei Teile:<br />

Der Aufgabenteil enthält zwei Aufgaben, die mit insgesamt 50 Punkten bewertet werden.<br />

Der Fragenteil enthält 25 Fragen, die mit jeweils zwei Punkten bewertet werden. Von jeweils drei Antworten<br />

ist nur eine richtig. Um Manipulationen auszuschließen, wird innerhalb des Fragenteils von der hier erreichten<br />

Punktzahl für jede falsch angekreuzte Antwort ein Punkt abgezogen. Statistisches Ankreuzen ergibt<br />

damit null Punkte. Es darf nur jeweils eine Antwort angekreuzt werden.<br />

Die Klausur ist mit 50 Punkten bestanden.<br />

w<br />

n<br />

v N<br />

v<br />

m<br />

Aufgabe 1: (25 Punkte)<br />

In einem Schallplattentonabnehmer findet sich die gezeichnete<br />

Anordnung mit<br />

m = 20 g, n = 0,02 m/N und w = 1 Ns/m<br />

a) Geben Sie die Übertragungsfunktion H= v N / v an.<br />

b) Bestimmen Sie die Frequenz der Tonarmresonanz aus<br />

m und n.<br />

c) Zeichnen Sie das Bode-Diagramm für |H| im Bereich<br />

0,1 f 0 ≤ f ≤ 10 f 0 , indem Sie die Asymptoten für f « f 0<br />

und f » f 0 bestimmen.<br />

d) Welchen Wert hat |H| bei f = f 0 ?<br />

Aufgabe 2: (25 Punke)<br />

Eine Werkstatt hat die Abmessungen 10m·7m·3m. der mittlere Schallabsorptionsgrad beträgt<br />

bei f = 500 Hz α= 5% und bei f = 1 kHz α= 7%.<br />

Eine Maschine erzeugt überwiegend in der Oktav mit fm = 500 Hz eine akustische Leistung<br />

Pak1 = 10mW , eine zweite Maschine in der Oktav mit fm = 1 kHz Pak2 = 15mW.<br />

Bestimmen Sie den A-bewerteten Gesamtschallpegel im diffusen Schallfeld, wenn beide Maschinen<br />

laufen.


Name: ______________________________________________ Matrikelnummer: ___________ - 2 -<br />

————————————————————————————————————————<br />

Fragenteil: WS 1998/99 E A<br />

1)<br />

2)<br />

3)<br />

4)<br />

5)<br />

6)<br />

7)<br />

8)<br />

9)<br />

10)<br />

11)<br />

12)<br />

13)<br />

Die Nachhallzeit eines Raums ist umgekehrt proportional zu<br />

± der Schallgeschwindigkeit ± der eingestrahlten Schalleistung ± dem Richtfaktor des<br />

Schallsenders<br />

Eine Schallwelle wird vollständig reflektiert, wenn die Schallkennimpedanz Z0 des Mediums, auf das der<br />

Schall trifft,<br />

± angepaßt ist ± schallweich ist ± reell ist.<br />

Der Frequenzgang eines dynamischen Mikrofons<br />

± ist im Prinzip sehr breitbandig<br />

± muß durch besondere Maßnahmen breitbandig gemacht werden<br />

± zeigt mehrere scharfe Resonanzspitzen<br />

Nach dem Schottkyschen Tiefenempfangsgesetz<br />

± ist es verboten, tiefe Frequenzen zu empfangen<br />

± kann man tiefe Frequenzen leichter empfangen als senden<br />

± kann man tiefe Frequenzen leichter senden als empfangen<br />

Die Phasengeschwindigkeit von Biegewellen in Platten ist<br />

± unabhängig von f ± proportional zu f ± proportional zu √f<br />

Die Wirkung einer elektrischen Feldkraft auf elektrische Ladungen in einem Kristallgitter wird beim<br />

± eletrostatischen ± elektromagnetischen ± piezoelektrischen Wandler genutzt<br />

Bei Messungen an einem Kolbenstrahler muß man einen bestimmten Mindestabstand einhalten, da im<br />

Nahfeld<br />

± Interferenzen auftreten ± Druck und Schnelle nicht in Phase sind ± der Schalldruck zu hoch ist<br />

Wirkt auf einen zylindrischen Körper eine Kraft senkrecht auf die Stirnfläche, beschreibt der<br />

Elastizitätsmodul E den Zusammenhang zwischen<br />

± Dehnung und Querkontraktion ± Kraft und Normalspannung ± Normalspannung und Dehnung<br />

An einem elektromechanischen Wandler berechnen Sie die Einheit einer Masse als<br />

± 1 Vm/Ws ± 1 As/m 2 ± 1 N 2 s/VA<br />

Markieren Sie die richtige Einheit.<br />

Der Artikulationsort ist<br />

± die Stelle im Vokaltrakt, wo ein Laut entsteht<br />

± eine Art Beichtstuhl<br />

± die Frequenzlage des ersten Formanten<br />

Ein der Empfindung der Lautstärke proportionales Maß ist<br />

± die Lautheit in sone ± der Lautstärkepegel in phon ± der Schalldruckpegel in dB<br />

Schallabsorption bei der Schallausbreitung in Luft beeinflußt die Nachhallzeit eines Raums<br />

± bei hohen Frequenzen ± bei tiefen Frequenzen ± gar nicht<br />

Die Masse nimmt im Kräftekreis eine Sonderstellung ein:<br />

± sie darf nicht mit anderen Elementen parallelgeschaltet werden<br />

± sie darf nicht mit anderen Elementen in Serie geschaltet werden<br />

± an ihr fällt Schnelle nur gegen Bezugserde ab


Name: ______________________________________________ Matrikelnummer: ___________ - 3 -<br />

————————————————————————————————————————<br />

Fragenteil: WS 1998/99 E A<br />

14)<br />

15)<br />

16)<br />

17)<br />

18)<br />

19)<br />

20)<br />

21)<br />

22)<br />

23)<br />

24)<br />

25)<br />

Unter Mithörschwelle versteht man den Schallpegel, bei dem Schall gerade<br />

± wahrgenommen ± unangenehm laut ± bei Anwesenheit von Störschall wahrgenommen<br />

wird<br />

Künstliche Sprache kann man erzeugen durch aneinanderreihen von<br />

± Phonemen ± Cepstren ± Vokalen<br />

Die akustische Leistung eines Kugelstrahlers erster Ordnung wird mit der eines Strahlers erster Ordnung bei<br />

gleicher Oberfläche und gleichem Ausschlag bei kR


FACHHOCHSCHULE KÖLN FACHBEREICH NACHRICHTENTECHNIK<br />

TECHNISCHE AKUSTIK<br />

KLAUSUR ANFANG WS 1998/99 AM 2. OKTOBER 1998<br />

Prof. Dr.-Ing. J.L Leichsenring<br />

Diese Klausur gliedert sich in zwei Teile:<br />

Der Aufgabenteil enthält zwei Aufgaben, die mit insgesamt 50 Punkten bewertet werden.<br />

Der Fragenteil enthält 25 Fragen, die mit jeweils zwei Punkten bewertet werden. Von jeweils drei<br />

Antworten ist nur eine richtig. Um Manipulationen auszuschließen, wird innerhalb des Fragenteils von<br />

der hier erreichten Punktzahl für jede falsch angekreuzte Antwort ein Punkt abgezogen. Statistisches<br />

Ankreuzen ergibt damit null Punkte. Es darf nur jeweils eine Antwort angekreuzt werden.<br />

Die Klausur ist mit 50 Punkten bestanden.<br />

reflexionfreier Abschluß<br />

Zak1<br />

Elektrodenabstand<br />

d<br />

S1<br />

S2<br />

l1 l2<br />

Membranfläche<br />

S<br />

Aufgabe 1: (25 Punkte)<br />

Gegeben ist ein Rohr mit Querschnittssprung in der<br />

gezeichneten Anordnung. Der rechte Rohrteil ist<br />

reflexionsfrei abgeschlossen.<br />

l 1 = 0,3 m; S 1 = 0,008 m 2 ; l 2 = 0,2 m; S 2 = 0,002 m 2<br />

c = 345 m/s; ρ 0 = 1,2 kg/m 3 ; f = 1,15 kHz<br />

Bestimmen Sie die akustische Impedanz Z ak1 , die man<br />

am linken Ende der Leitung findet.<br />

Aufgabe 2: (25 Punke)<br />

Für ein Kondensatormikrofon als Druckempfänger ist<br />

gegeben:<br />

wirksame Membranfläche S = 2 cm 2<br />

wirksame Membranmasse m = 20 mg<br />

Vorspannung U = 100 V<br />

mech. Resonanzfrequenz f 0 = 10 kHz<br />

Gesucht ist der Abstand der Membran zur<br />

Gegenelektrode, wenn ein<br />

Leerlauf-Druckübertragungsfaktor von T p = 5 mV/Pa


Name: ______________________________________________ Matrikelnummer: ___________ - 3 -<br />

————————————————————————————————————————<br />

Fragenteil: WS 1998/99 A<br />

————————————————————————————————————————<br />

01) Die empfundene Lautheit halbiert sich, wenn der Lautstärkepegel um<br />

± 10 dB ± 3 dB ± 1 dB gesenkt wird<br />

————————————————————————————————————————————————<br />

02) Akustische Eigenschwingungen in einem Rohr, das an einer Seite schallhart, an der anderen Seite<br />

reflexionsfrei abgeschlossen ist, treten auf, wenn die Länge des Rohrs ein Vielfaches<br />

± der halben Wellenlänge ist,<br />

± einer viertel Wellenlänge ist.<br />

± Es treten keine Eigenschwingungen auf.<br />

————————————————————————————————————————————————<br />

03) Die Schallenergie je Volumeneinheit bezeichnet man als<br />

± Schallenergiedichte ± Schallintensität ± Schallpegel<br />

————————————————————————————————————————————————<br />

04) Der Absorptionsgrad an einer Grenzfläche ist definiert als das Verhältnis der<br />

± Schallenergiedichten des absorbierten und hinlaufenden Schalls<br />

± Schalldrücke des hinlaufenden und reflektierten Schalls<br />

± Schalldrücke des absorbierten und hinlaufenden Schalls<br />

————————————————————————————————————————————————<br />

05) Die Spektren Zweier Geräusche, liegen in einer Frequenzgruppe. Treten beide gleichzeitig auf,<br />

so kann man ihre Lautheit berechnen aus der Summe der<br />

± Lautstärken ± Intensitäten ± Einzellautheiten<br />

der einzelnen Geräusche.<br />

————————————————————————————————————————————————<br />

06) Die Gesamtmasse zweier parallelgeschalteter Massen ergibt sich als<br />

± 1/m = 1/m1+ 1/m2<br />

± m = m1+ m2<br />

± Die Parallelschaltung ist nicht möglich.<br />

————————————————————————————————————————————————<br />

07) Als Morphene bezeichnet man<br />

± Schmerzmittel<br />

± spezielle Kuststoffe zur Membranherstellung<br />

± Bausteine der Sprache<br />

————————————————————————————————————————————————<br />

08) An einem Querschnittssprung in einem Rohr verhalten sich die<br />

± spezifischen ± akustischen ± mechanischen<br />

Inpedanzen wie die Querschnittsflächen<br />

————————————————————————————————————————————————<br />

09) Oberhalb seiner Grenzfrequenz (k > g) überträgt ein Exponentialtrichter unendlicher Länge<br />

± keine ± wenig ± sehr gut akustische Wirkleistung<br />

————————————————————————————————————————————————<br />

10) Fällt bei einem Kondensatormikrofon die Polarisationsspannung aus, mißt man an den Klemmen der<br />

Mikrofonkapsel eine Signalspannung, die<br />

± dem Schalldruck proportional ist<br />

± dem Quadrat des Schalldrucks proportional ist<br />

± gleich Null ist<br />

————————————————————————————————————————————————<br />

11) Die mechanische und die elektrische Impedanz sind bei einem elektrostatischen Wandler über<br />

± M ± N 2 ± M 2 miteinander verknüpft<br />

————————————————————————————————————————————————<br />

12) Den Pegel, bei dem ein Ton bei gleichzeitiger Darbietung eines Geräuschs gerade hörbar wird, ist die<br />

± Hörschwelle ± Mithörschwelle ± Fühlschwelle<br />

————————————————————————————————————————————————<br />

13) In einem schlanken runden Stab werden Biegewellen angeregt, ihre Phasengeschwindigkeit<br />

± ist proportional zur Stablänge<br />

± hängt nicht von den Abmessungen ab<br />

± wächst mit steigendem Stabdurchmesser<br />

————————————————————————————————————————————————


Name: ______________________________________________ Matrikelnummer: ___________ - 4 -<br />

————————————————————————————————————————<br />

Fragenteil: WS 1998/99 A<br />

————————————————————————————————————————<br />

14) Als Schallfluß bezeichnet man das Produkt aus<br />

± Schnelle v und Druck p im Schallfeld<br />

± Schnelle v und Querschnittsfläche S in akustischen Filtern<br />

± Intensität I und Schallgeschwindigkeit c im Schallfeld<br />

————————————————————————————————————————<br />

15) Für eine gute Verständlichkeit von Sprache in einem Raum<br />

± muß die Nachhallzeit sehr groß sein<br />

± gibt es ein Optimum der Nachhallzeit<br />

± kann die Nachhallzeit beliebige Werte annehmen<br />

————————————————————————————————————————<br />

16) Die von einem Druckgradientenempfänger abgegebene Spannung ist proportional<br />

± zur Schallschnelle ± zum Schalldruck ± zum Ausschlag im Schallfeld<br />

————————————————————————————————————————<br />

17) Die Kraft an einer Feder ist proportional zu<br />

± zur Auslenkung ± zur Beschleunigung ± zur Schnelle<br />

————————————————————————————————————————<br />

18) Eine Mikrofon mit Hypernierencharakteristik hat eine Hauptkeule, die<br />

± schmaler ± breiter ± gleich<br />

ist im Vergleich zur einfachen Nierencharakteristik<br />

————————————————————————————————————————<br />

19) Die Reflexion eines Schallimpulses trifft bei einem Hörer mit mehr als 50 ms Verzögerung ein.<br />

Dabei ± steigt die Tonhöhe an<br />

± hört man ein Echo<br />

± verschmelzen beide Höreindrücke<br />

————————————————————————————————————————<br />

20) An der Oberfläche eines Kristalls werden Rayleighwellen erzeugt. Bei diesen Wellen<br />

± schwingt nur die Oberfläche<br />

± nimmt die Amplitude von er Oberfläche nach innen exponentiell ab<br />

± wird der ganze Körper von den Wellen gleichmäßig durchdrungen<br />

————————————————————————————————————————<br />

21) Am Ausgang eines Terzfiltersatzes soll in jedem Bereich der gleiche Effektivwert der Spannung gemessen<br />

werden. Dazu muß man am Eingang<br />

± "rosa" ± weißes ± oktavgefiltertes Rauschen angelegen.<br />

————————————————————————————————————————<br />

22) Wenn man synthetische Sprache durch Aneinanderfügen von Phonemen als<br />

Sprachbausteine erzeugt, hat diese<br />

± sehr gute ± mäßige ± ungenügende Qualität.<br />

————————————————————————————————————————<br />

23) Unter Schallschnelle versteht man die<br />

± Phasengeschwindigkeit des Schalls<br />

± Strömungsgeschwindigkeit der Materie<br />

± Schwinggeschwindigkeit eines Materieteilchens<br />

————————————————————————————————————————<br />

24) In einem Richtdiagramm z.B. eines Lautsprechers trägt man<br />

± die Linien gleichen Schalldrucks<br />

± den Schalldruck oder Schallpegel in Polarkoordinaten<br />

± die Linien gleicher Schallintensität auf.<br />

————————————————————————————————————————<br />

25) In einem schlanken runden Stab werden Torsionswellen angeregt, ihre Phasengeschwindigkeit<br />

± hängt nicht von den Abmessungen ab<br />

± wächst mit steigendem Stabdurchmesser<br />

± ist proportional zur Stablänge


FACHHOCHSCHULE KÖLN FACHBEREICH NACHRICHTENTECHNIK<br />

TECHNISCHE AKUSTIK<br />

KLAUSUR ENDE SS 1998 AM 26. JUNI 1998 A<br />

Prof. Dr.-Ing. J.L Leichsenring<br />

————————————————————————————————————————<br />

Diese Klausur gliedert sich in zwei Teile:<br />

Der Aufgabenteil enthält zwei Aufgaben, die mit insgesamt 50 Punkten bewertet werden.<br />

Der Fragenteil enthält 25 Fragen, die mit jeweils zwei Punkten bewertet werden. Von jeweils drei<br />

Antworten ist nur eine richtig. Um Manipulationen auszuschließen, wird innerhalb des Fragenteils<br />

von der hier erreichten Punktzahl für jede falsch angekreuzte Antwort ein Punkt abgezogen.<br />

Statistisches Ankreuzen ergibt damit null Punkte. Es darf nur jeweils eine Antwort angekreuzt<br />

werden.<br />

Die Klausur ist mit 50 Punkten bestanden.<br />

————————————————————————————————————————<br />

Bei allen Aufgaben ist die Lösung zu entwickeln. Bloße Ergebniswerte oder Ergebnisse unter<br />

Verwendung von Formeln fremder Herkunft gelten nicht als Lösung.<br />

————————————————————————————————————————<br />

2cm<br />

4cm<br />

2cm 2cm<br />

1cm<br />

Aufgabe 2: (25 Punkte)<br />

Aufgabe 1: (25 Punkte)<br />

Die Membran eines elektroakustischen erzeugt<br />

einen Schallfluß q , der ein akustisches Filter<br />

1<br />

durchläuft, an dessen Ende man q findet.<br />

2<br />

a) Zeichnen Sie das akustische Ersatzbild des<br />

Filters und bestimmen Sie die Bauelemente.<br />

b) Bestimmen Sie die Übertragungsfunktion<br />

|H (f )| = q /q . 2 1<br />

c) Bei welcher Frequenz f tritt Resonanz auf ?<br />

0<br />

d) Zeichnen Sie den prinzipiellen Verlauf der<br />

Funktion |H (f /f )| im Bodediagramm für<br />

0<br />

0,01f ≤ f ≤ 100f .<br />

0 0<br />

Die Wände eines Raums mit den Abmessungen 4m⋅3m⋅2,7m haben einen mittleren<br />

Schallabsorptionsgrad α = 0,15. Vor einem Sprecher (Kugelcharakteristik) stehen im Abstand<br />

r = 0,5 m ein Kugel- und ein Nierenmikrofon (Bündelungsgrad γ = 3, auf den Sprecher<br />

gerichtet).<br />

Bestimmen Sie die Pegelunterschiede zwischen direktem und diffusem Schallanteil in den von<br />

beiden Mikrofonen aufgenommenen Signalen.<br />

Rechnen Sie mit c = 345 m/s und ρ 0 = 1,2 kg/m 3


Name: ______________________________________________ Matrikelnummer: ___________ - 2 -<br />

————————————————————————————————————————<br />

Fragenteil: SS 1998 A<br />

————————————————————————————————————————<br />

01) Schallflüsse in einer akustischen Schaltung berechnet man<br />

± nach der Maschenregel ± nach der Knotenregel ± mal so, mal anders<br />

————————————————————————————————————————————————<br />

02) Die von uns benutzte Wellengleichung für Schallwellen in Luft gilt für<br />

± ruhende Luft ohne Leitungsverluste<br />

± strömende Luft ohne Leitungsverluste<br />

± ruhende Luft mit Leitungsverlusten<br />

————————————————————————————————————————————————<br />

03) Die Ausbreitungsrichtung der Welle und die Schallschnelle stehen bei<br />

± Transversalwellen ± Dichtewellen ± Dehnwellen<br />

aufeinander senrecht<br />

————————————————————————————————————————————————<br />

04) Bei weißem Rauschen ist der lineare Mittelwert des Schalldrucks gleich<br />

± Null ± seinem Effektivwert ± seinem Scheitelwert<br />

————————————————————————————————————————————————<br />

05) Die Ordnungszahl eines Strahlers ergibt sich aus<br />

± der Zahl der Knotenlinien auf seiner Oberfläche<br />

± der Form des Strahlers<br />

± der Größe seiner Oberfläche<br />

————————————————————————————————————————————————<br />

06) Die Schallschnelle im Kugelschallfeld nullter Ordnung<br />

± hat eine Tangential- und eine Normalkomponente<br />

± ist immer tangential zu konzentrischen Kugelflächen<br />

± ist immer senkrecht zu konzentrischen Kugelflächen<br />

————————————————————————————————————————————————<br />

07) In einem Richtdiagramm z.B. eines Lautsprechers trägt man<br />

± die Linien gleichen Schalldrucks<br />

± die Linien gleicher Schallintensität<br />

± den Schalldruck oder Schallpegel in Polarkoordinaten auf.<br />

————————————————————————————————————————————————<br />

08) Die Stelle im Vokaltrakt, wo ein Laut geformt wird, bezeichnet man als<br />

± Stimmritze ± Formant ± Artikulationsort<br />

————————————————————————————————————————————————<br />

09) Unter Biegewellen versteht man<br />

± eine neue Form elastischer Antriebswellen beim PKW<br />

± eine Schwingungsform in Stäbenn und Platten<br />

± Lügen, bei denen sich die Balken biegen<br />

————————————————————————————————————————————————<br />

10) In einer stehenden Welle verhalten sich die Schalldrücke in den Minima und Maxima wie 1:3, der<br />

Reflexionsfaktor ist dann<br />

± 0,333 ± 0,5 ± 1,5<br />

————————————————————————————————————————————————<br />

11) In einem Hallraum mit ideal gut reflektierenden Wänden wird die Nachhallzeit<br />

± beliebig groß ± endlich groß ± beliebig klein<br />

————————————————————————————————————————————————<br />

12) Liegen die Frequenzen zweier Geräusche in unterschiedlichen Frequenzgruppen, kann man ihre<br />

Gesamtlautstärke berechnen aus der Summe der<br />

± Lautstärken ± Intensitäten ± Lautheiten<br />

der Geräuschanteile in den beiden Frequenzgruppen.<br />

————————————————————————————————————————————————<br />

13) Die Mündungskorrektur für ein offenes Rohrende berücksichtigt, daß sich das Rohr bez. seiner Eigentöne<br />

verhält, als sei es<br />

± etwas kürzer ± etwas länger ± viel länger<br />

als seine tatsächliche Länge beträgt.<br />

————————————————————————————————————————————————


Name: ______________________________________________ Matrikelnummer: ___________ - 3 -<br />

————————————————————————————————————————<br />

Fragenteil: SS 1998 A<br />

————————————————————————————————————————<br />

14) A-bewerteter Schallpegel ist der Schallpegel,<br />

± der ohne Frequenzbewertung gemessen wird<br />

± der der vom menschlichen Ohr empfunden wird<br />

± dessen Frequenzgang mit der genormten A-Kurve bewertet wurde<br />

————————————————————————————————————————————————<br />

15) Unter Schallschnelle versteht man :<br />

± die Ausbreitungsgeschwindigkeit des Schalls<br />

± die Reaktionsgeschwindigkeit auf einen akustischen Reiz<br />

± die Geschwindigkeit eines Materieteilchens<br />

————————————————————————————————————————————————<br />

16) Die akustische Masse in einem Rohrstück wird bei gleicher Länge und abnehmender Querschittsfläche<br />

± größer ± kleiner ± bleibt gleich<br />

————————————————————————————————————————————————<br />

17) Um bei einem Kondensatormikrofon als Druckempfänger eine niedrige untere Grenzfrequenz zu erhalten,<br />

muß ± die Membran sehr groß sein<br />

± die Vorspannung hoch sein<br />

± der Lastwiderstand hochohmig sein<br />

————————————————————————————————————————————————<br />

18) Zwei parallelgeschaltete akustische Massen kann man ersetzen durch eine akustische Gesamtmasse mit<br />

± 1/M ak = 1/M ak1 + 1/M ak2<br />

± M ak = M ak1 + M ak2<br />

± die Zusammenfassung ist nicht möglich<br />

————————————————————————————————————————————————<br />

19) Der Quotient Kraft/Schnelle beschreibt eine<br />

± spezifische Schallfeldimpedanz<br />

± mechanische Impedanz<br />

± akustische Impedanz<br />

————————————————————————————————————————————————<br />

20) Die Hauptkeule der Richtcharakteristik eines Mikrofons mit Supernierencharakteristik ist<br />

± gleich breit ± breiter ± schmaler<br />

verglichen mit einer einfachen Nierencharakteristik.<br />

————————————————————————————————————————————————<br />

21) Die Resonanzfrequenz eines Tieftonlautsprechers in einem Baßreflexgehäuse liegt<br />

± in der Nähe des unteren Endes<br />

± in der Mitte<br />

± am oberen Ende seines Übertragungsbereichs<br />

————————————————————————————————————————————————<br />

22) Unter Artikulation versteht man ± die Formung von Sprachlauten<br />

± das Verfassen eines Zeitungsartikels<br />

± eine Sprachverständlichkeitsmessung<br />

————————————————————————————————————————————————<br />

23) Eine Halbierung des Lautstärkeeindrucks eines Geräuschs mit 73 dB Schallpegel, erhält man, wenn man<br />

den Pegel auf<br />

± 70 dB ± 67 dB ± 63 dB reduziert<br />

————————————————————————————————————————————————<br />

24) In einem Kugelschallfeld nullter Ordnung<br />

± ist es im Nahfeld besonders ruhig<br />

± überwiegt im Nahfeld die Blindleistung<br />

± treten im Nahfeld Interferenzen auf<br />

————————————————————————————————————————————————<br />

25) Die Richtcharakteristik eines Kugelstrahlers 1. Ordnung ist<br />

± eine Kugel ± eine Acht ± eine Niere<br />

————————————————————————————————————————————————


FACHHOCHSCHULE KÖLN FACHBEREICH NACHRICHTENTECHNIK<br />

TECHNISCHE AKUSTIK<br />

KLAUSUR WS 1997/98 E AM 9. FEBRUAR 1998 A<br />

Prof. Dr.-Ing. J.L Leichsenring<br />

————————————————————————————————————————<br />

Diese Klausur gliedert sich in zwei Teile:<br />

Der Aufgabenteil enthält zwei Aufgaben, die mit insgesamt 50 Punkten bewertet werden.<br />

Der Fragenteil enthält 25 Fragen, die mit jeweils 2 Punkten bewertet werden. Von jeweils 3<br />

Antworten ist nur eine richtig. Um Manipulationen auszuschließen, wird innerhalb des Fragenteils<br />

von der hier erreichten Punktzahl für jede falsch angekreuzte Antwort ein Punkt abgezogen.<br />

Statistisches Ankreuzen ergibt damit null Punkte. Es darf nur jeweils eine Antwort angekreuzt<br />

werden.<br />

Die Klausur ist mit 50 Punkten bestanden.<br />

————————————————————————————————————————<br />

Bei allen Aufgaben ist die Lösung zu entwickeln. Bloße Ergebniswerte oder Ergebnisse unter<br />

Verwendung von Formeln fremder Herkunft gelten nicht als Lösung.<br />

————————————————————————————————————————<br />

Aufgabe 1: (25 Punkte) Ein Lautsprecher in einem geschlossenen Gehäuse hat einen<br />

Membrandurchmesser D = 5 cm. Er gibt im freien Schallfeld in<br />

einem Abstand R = 1m bei f = 150 Hz einen Wirk-Schallpegel L<br />

= 90 dB ab.<br />

Welche Blindleistung muß der Lautsprecher an das Schallfeld<br />

abgeben?<br />

Aufgabe 2: (25 Punkte) Für ein dynamisches Mikrofon als Druckempfänger ist gegeben:<br />

Membrandurchmesser D = 2,5 cm<br />

Luftspaltinduktion B = 1 T<br />

Schwingspuldurchmesser D SP = 2 cm<br />

Wicklungswiderstand R 1 = 200 W<br />

Die Induktivität der Schwingspule wird vernachlässigt.<br />

Wieviele Windungen muß die Schwingspule haben, wenn in der<br />

Mitte des Übertragungsbereichs der Leerlauf-Druck-<br />

Übertragunsfaktor T p = 1 mV/Pa betragen soll?<br />

Hier mißt man am Mikrofon im Übrtagungsbereich bei<br />

frei beweglicher Schwingspule einen Widerstand<br />

R ges = 210 W.<br />

Rechnen Sie mit c = 340 m /s und ρ o = 1,2 kg /m 3


Name: ______________________________________________ Matrikelnummer: ___________ - 2 -<br />

————————————————————————————————————————<br />

Fragenteil: WS 1997/98 E A<br />

————————————————————————————————————————<br />

01) Verwendet man als Druckgradientenempfänger ein dynamisches Mikrofon,<br />

muß die Resonanzfrequenz<br />

± in der Mitte ± am unteren Ende ± am oberen Ende<br />

des Übertragungsbereichs liegen.<br />

————————————————————————————————————————<br />

02) Der Übertragungsbereich bei elektrodynamischen Tieftonlautsprechern liegt<br />

± oberhalb ± unterhalb ± beiderseits<br />

ihrer mechanischen Resonanzfrequenz.<br />

————————————————————————————————————————<br />

03) Ein Vorverstärker für ein Kondensatormikrofon muß einen<br />

± hochohmigen ± niederohmigen ± kapazitiven<br />

Eingangswiderstand besitzen.<br />

————————————————————————————————————————<br />

04) Eine Schallwelle wird vollständig absorbiert, wenn die Schallkennimpedanz Z 0 des Mediums,<br />

auf das der Schall trifft<br />

± schallweich ± angepaßt ± schallhart ist.<br />

————————————————————————————————————————<br />

05) Die Schallgeschwindigkeit von Dehnwellen in runden Stäben<br />

± wächst mit steigendem Stabdurchmesser<br />

± ist proportional zur Stablänge<br />

± hängt nicht von den Abmessungen ab.<br />

————————————————————————————————————————<br />

06) Interferenzen findet man im Nahfeld von<br />

± Kolbenstrahlern nullter Ordnung<br />

± Kugelstrahlern nullter Ordnung<br />

± Kugelstrahlern erster Ordnung<br />

————————————————————————————————————————<br />

07) Den Schalldruck kann man beschreiben durch einen<br />

± räumlichen Vektor ± konstanten Wert ± Skalar<br />

————————————————————————————————————————<br />

08) Ein Allophon ist ± ein drahtloses Telephon<br />

± eine Einheit für die Lautstärke<br />

± ein Baustein der Sprache<br />

————————————————————————————————————————<br />

09) Bestimmen Kraftwirkungen im elektrischen Feld den Wandlereffekt eines elektromechanischen Wandlers,<br />

so liegt ein<br />

± Relaiswandler ± M-Wandler ± N-Wandler vor.<br />

————————————————————————————————————————<br />

10) Auf einem schlanken Stab mit quadratischem Querschnitt breitet sich eine Biegewelle aus.<br />

Bei steigender Frequenz<br />

± steigt die Ausbreitungsgeschwindigkeit.<br />

± fällt die Ausbreitungsgeschwindigkeit<br />

± ändert sich die Ausbreitungsgeschwindigkeit nicht<br />

————————————————————————————————————————<br />

11) Schaltet man zwei gleiche mechanische Federn parallel, erhält man die<br />

± doppelte ± gleiche ± halbe Nachgiebigkeit einer einzelnen Feder<br />

————————————————————————————————————————<br />

12) In einem Quaderraum wird ein impulsförmiger Schall erzeugt.<br />

Die Zahl der bei einem Mikrofon je Zeiteinheit eintreffenden Schallrückwürfe steigt mit der<br />

± ersten ± zweiten ± dritten Potenz der Zeit an.<br />

————————————————————————————————————————<br />

13) Eine ideal starre mechanische Verbindung hat die Impedanz<br />

± undendlich ± Null ± einer weichen Feder<br />

————————————————————————————————————————


Name: ______________________________________________ Matrikelnummer: ___________ - 3 -<br />

————————————————————————————————————————<br />

Fragenteil: WS 1997/98 E A<br />

————————————————————————————————————————<br />

14) Bei einem Druckgradientenempfänger wirkt das Schallfeld<br />

± auf eine Seite der Membran voll, auf die andere teilweise<br />

± auf beide Seiten der Membran<br />

± nur auf eine Seite der Membran.<br />

————————————————————————————————————————<br />

15) Untersuchungen über die Eigenschaften des Gehörs haben ergeben, daß im Ohr<br />

± nie ± bei hohen Pegeln ± bei niedrigen Pegeln<br />

Verzerrungen auftreten<br />

————————————————————————————————————————<br />

16) Die von uns benutzte Wellengleichung für Schallwellen in Luft gilt für<br />

± ruhende Luft mit Leitungsverlusten<br />

± strömende Luft ohne Leitungsverluste<br />

± ruhende Luft ohne Leitungsverluste<br />

————————————————————————————————————————<br />

17) Ein Nierenmikrofon hat bei Wellenlängen, die viel kleiner als seine Abmessungen sind<br />

± eine frequenzunabhängige Nierencharakteristk<br />

± eine Kugelcharakteristik<br />

± eine frequenzabhängige Richtcharakteristik<br />

————————————————————————————————————————<br />

18) Unter Schallgeschwindigkeit versteht man die<br />

± Höchstgeschwindigkeit von Verkehrsflugzeugen<br />

± Phasengeschwindigkeit einer Schallwelle<br />

± Momentangeschwindigkeit der Luftmoleküle<br />

————————————————————————————————————————<br />

19) Eine Schallquelle erzeugt am Meßort einen Ton mit dem Schalldruckpegel 60 dB bei der Frequenz 50Hz.<br />

Wenn man das Meßgerät auf "A-bewerteter Schallpegel" umschaltet, zeigt es<br />

± genau ± mehr als ± weniger als 60 dBA an.<br />

————————————————————————————————————————<br />

20) Schall breite sich in einem Rohr mit Querschnittssprung aus.<br />

Unmittelbar vor und hinter dem Querschnittssprung sind die<br />

± Schallschnellen ± Schallflüsse ± spezifischen Impedanzen gleich<br />

————————————————————————————————————————<br />

21) Als akustische Masse bezeichnet man<br />

± die flächenbezogene Masse in einem Rohrstück<br />

± einen besonders standfesten Lautsprecher<br />

± eine schwergewichtige Sängerin<br />

————————————————————————————————————————<br />

22) Sollen Vokale erkannt werden, so kommt es wesentlich auf<br />

± die Tonhöhe der Stimme an<br />

± die Hüllkurve des Leistungsdichtespektrums an<br />

± eine streng periodische Anregung der Stimmbänder an<br />

————————————————————————————————————————<br />

23) Will man auf einem Kanal mehr als 99% des Zeitverlaufs eines rauschähnlichen Signals unverfälscht<br />

übertragen, muß der Kanal Spitzenwerte vom<br />

± 1 fachen ± 3 fachen ± 10 fachen<br />

Effektivwert unverzerrt übertragen können.<br />

————————————————————————————————————————<br />

24) Die genäherte Beschreibung eines Kolbenstrahlers durch einen äquivalenten Kugelstrahler ist schlecht bei<br />

± hohen Frequenzen ± kR=1 ± tiefen Frequenzen<br />

————————————————————————————————————————<br />

25) Die Schallausbreitung in Luft ist<br />

± auf jeden Fall ein linearer Vorgang<br />

± für kleine Amplituden genähert linear<br />

± für tiefe Frequenzen nichtlinear<br />

————————————————————————————————————————


FACHHOCHSCHULE KÖLN FACHBEREICH NACHRICHTENTECHNIK<br />

TECHNISCHE AKUSTIK<br />

KLAUSUR WS 1997/98 AM 2. OKTOBER 1997 A<br />

Prof. Dr.-Ing. J.L Leichsenring<br />

————————————————————————————————————————<br />

Diese Klausur gliedert sich in zwei Teile:<br />

Der Aufgabenteil enthält zwei Aufgaben, die mit insgesamt 50 Punkten bewertet werden.<br />

Der Fragenteil enthält 25 Fragen, die mit jeweils 2 Punkten bewertet werden. Von jeweils 3<br />

Antworten ist nur eine richtig. Um Manipulationen auszuschließen, wird innerhalb des Fragenteils<br />

von der hier erreichten Punktzahl für jede falsch angekreuzte Antwort ein Punkt abgezogen.<br />

Statistisches Ankreuzen ergibt damit null Punkte. Es darf nur jeweils eine Antwort angekreuzt<br />

werden.<br />

Die Klausur ist mit 50 Punkten bestanden.<br />

————————————————————————————————————————<br />

Bei allen Aufgaben ist die Lösung zu entwickeln. Bloße Ergebniswerte oder Ergebnisse unter<br />

Verwendung von Formeln fremder Herkunft gelten nicht als Lösung.<br />

————————————————————————————————————————<br />

Aufgabe 1: (25 Punkte) In einem Raum mit den Abmessungen 10m 4,7m 2,7m und<br />

der Nachhallzeit T = 1,7 s erzeugt eine Maschine einen<br />

Schallpegel L = 97 dB.<br />

a) Bestimmen Sie die von der Maschine abgegebene akustische<br />

Leistung.<br />

b) Sie haben die Aufgabe, den Schallpegel auf L = 60 dB zu<br />

senken. In einem ersten Ansatz soll geprüft werden, ob dies<br />

durch eine Erhöhung der Wandabsorption zu erreichen ist.<br />

Bestimmen Sie hierzu den erforderlichen Absorptionsgrad der<br />

als gleich absorbierend angenommenen Wände (inkl.<br />

Decke und Boden) und diskutieren Sie die technische<br />

Realisierbarkeit.<br />

Aufgabe 2: (25 Punkte) Die Oktavpegelmessung eines Geräuschs ist in folgender<br />

Tabelle zusammengestellt:<br />

f /Hz m Oktav 250 500 1k 2k 4k<br />

L / dB<br />

oktav<br />

40 83 65 55 40<br />

a) Berechnen Sie den unbewerteten Gesamtschallpegel des<br />

Geräuschs.<br />

b) Berechnen Sie die Lautheit und den Lautstärkepegel nach<br />

Stevens gemäß Übungsblatt 8. Interpolieren sie linear<br />

innerhalb der dort gegebenen Tabelle.


Name: ______________________________________________ Matrikelnummer: ___________ - 2 -<br />

————————————————————————————————————————<br />

Fragenteil: WS 1997/98 A<br />

————————————————————————————————————————<br />

01) Die Kraft an einer Feder ist proportional<br />

± zur Schnelle ± zum Ausschlag ± zur Beschleunigung<br />

————————————————————————————————————————————————<br />

02) Die Schallintensitätsdichte ist definiert als<br />

± Schallenergie/Volumen<br />

± Schalleistung/(Fläche·Hz)<br />

± Schalleistung/Volumen<br />

————————————————————————————————————————————————<br />

03) Die normierende Größe bei der Berechnung des Schallintensitätspegels ist<br />

± 10 -12 W/m2 ± 1W ± 2·10 -5 W/m2<br />

————————————————————————————————————————————————<br />

04) Wird ein Sinuston bei Anwesenheit eines Geräuschs dargeboten, so muß er einen bestimmten Pegel<br />

überschreiten, um hörbar zu werden. Diesen Pegel nennt man<br />

± Mithörschwelle ± Hörschwelle ± Fühlschwelle<br />

————————————————————————————————————————————————<br />

05) Ein Kugelstrahler 1. Ordnung hat die Richtcharakteristik einer<br />

± Acht ± Kugel ± Niere<br />

————————————————————————————————————————————————<br />

06) Am Querschittssprung zweier Rohre mit unterschiedlichem Durchmesser findet man den Reflexionsfaktor<br />

Null, wenn hier die<br />

± mechanischen ± spezifischen ± akustischen Impedanzen gleich sind.<br />

————————————————————————————————————————————————<br />

07) Bei einem Hörer in einem Raum treffen Rückwürfe eines Schallereignisses ein. Ihre Zahl<br />

± steigt linear mit der Zeit<br />

± fällt mit steigender Zeit<br />

± steigt mit der dritten Potenz der Zeit<br />

————————————————————————————————————————————————<br />

08) Ein Formant ist ein<br />

± Mitteltonlautsprecher<br />

± Mikrofoneichgerät<br />

± hervortretender Frequenzbereich bei Sprache<br />

————————————————————————————————————————————————<br />

09) Als Lösungen der Schallwellengleichung sind möglich<br />

± nur sinusförmige Zeitfunktionen<br />

± nur Sprach- und Musiksignale<br />

± beliebige Zeitfunktionen<br />

————————————————————————————————————————————————<br />

10) Bei Dehnwellen in Stäben<br />

± wird der Stab geringfügig verbogen<br />

± bewegt sich die Staboberfläche auch in Richtung senkrecht zur Stabachse<br />

± bewegt sich die Staboberfläche nur in Richtung der Stabachse<br />

————————————————————————————————————————————————<br />

11) Der Absorptionsgrad α von verputztem Mauerwerk liegt zwischen<br />

± 1% und 5% ± 10% und 50% ± 80% und 90%<br />

————————————————————————————————————————————————<br />

12) Werden beide Seiten einer kleinen Membran dem Schallfeld ausgesetzt, ist die resultierende Kraft auf<br />

die Membran proportional<br />

± dem Druckgradienten ± dem Druck ± gleich null<br />

————————————————————————————————————————————————


Name: ______________________________________________ Matrikelnummer: ___________ - 3 -<br />

————————————————————————————————————————<br />

Fragenteil: WS 1997/98 A<br />

————————————————————————————————————————<br />

13) In einem Stab mit quadratischem Querschnitt breiten sich Dehnwellen aus. Ihre Geschwindigkeit<br />

± steigt ± fällt ± ändert sich nicht bei steigender Frequenz<br />

————————————————————————————————————————————————<br />

14) Das menschliche Gehör kann im Bereich zwischen Hörschwelle und Fühlschwelle bei mittleren Frequenzen<br />

etwa<br />

± 12 ± 120 ± 1200 Stufen unterscheiden.<br />

————————————————————————————————————————————————<br />

15) Die mechchanische Impedanz wird bestimmt als<br />

± F/v ± v/F ± p/q<br />

————————————————————————————————————————————————<br />

16) Akustische Schaltelemente und Kreise werden<br />

± für akustische Filterschaltungen benötigt<br />

± möglichst vermieden<br />

± zur Schallfeldberechnung verwendet<br />

————————————————————————————————————————————————<br />

17) Ein Tieftonlautsprecher als Kugelstrahler nullter Ordnung arbeitet üblicherweise im Bereich<br />

± kR < 1 ± kR > 1 ± kR > 1 und kR < 1 wird überstrichen<br />

————————————————————————————————————————————————<br />

18) Die untere Grenzfrequenz eines Exponentialtrichters hängt ab<br />

± von seiner Länge<br />

± vom Wuchsmaß g des Trichterdurchmessers<br />

± von der Schnelletransformation beim Antriebssystem<br />

————————————————————————————————————————————————<br />

19) In einer mechanische Schaltung gilt für die Kräfte<br />

± die Maschenregel ± die Knotenregel ± keine der beiden Regeln<br />

————————————————————————————————————————————————<br />

20) Ein Mikrofon mit "Achtercharakteristik" ist näherungsweise ein<br />

± Druckempfänger ± Schnelleempfänger ± Intensitätsempfänger<br />

————————————————————————————————————————————————<br />

21) zur Beschreibung des Übertragungsverhaltens eines idealen elektromechnischen Wandlers benötigt man<br />

± drei ± zwei ± eine Größe/ Größen.<br />

————————————————————————————————————————————————<br />

22) Man kann die Größen der mechanischen Ersatzschaltung eines elektroakustischen Wandlers auf die<br />

elektrische Seite transformieren. Dabei ist die Zuordnung<br />

± im Sinne einer Analogie frei wählbar<br />

± durch die Art des inneren Wandlers vorgegeben<br />

± immer schaltungstreu<br />

————————————————————————————————————————————————<br />

23) Der Lautstärkeeindruck eines Geräuschs wird durch eine eigene Größe beschrieben. Deren Einheit ist<br />

± 1 dB ± 1 phon ± 1 sone<br />

————————————————————————————————————————————————<br />

24) Das Richtdiagramm Schallsenders zeigt<br />

± die Linien gleichen Schalldrucks<br />

± die Linien gleicher Schallintensität<br />

± den Schalldruck oder Schallpegel in Polarkoordinaten<br />

————————————————————————————————————————————————<br />

25) Die Sabine'sche Nachhallformel liefert für kleine Werte von α<br />

± richtige Nachhallzeiten<br />

± zu kurze Nachhallzeiten<br />

± zu lange Nachhallzeiten<br />

————————————————————————————————————————————————


FACHHOCHSCHULE KÖLN FACHBEREICH NACHRICHTENTECHNIK<br />

TECHNISCHE AKUSTIK<br />

KLAUSUR SS 1995 AM 20. MÄRZ 1997 A<br />

Prof. Dr.-Ing. J.L Leichsenring<br />

————————————————————————————————————————<br />

Diese Klausur gliedert sich in zwei Teile:<br />

Der Aufgabenteil enthält zwei Aufgaben, die mit insgesamt 50 Punkten bewertet werden.<br />

Der Fragenteil enthält 25 Fragen, die mit jeweils zwei Punkten bewertet werden. Von jeweils drei<br />

Antworten ist nur eine richtig. Um Manipulationen auszuschließen, wird innerhalb des Fragenteils<br />

von der hier erreichten Punktzahl für jede falsch angekreuzte Antwort ein Punkt abgezogen.<br />

Statistisches Ankreuzen ergibt damit null Punkte. Es darf nur jeweils eine Antwort angekreuzt<br />

werden.<br />

Die Klausur ist mit 50 Punkten bestanden.<br />

————————————————————————————————————————<br />

Bei allen Aufgaben ist die Lösung zu entwickeln. Bloße Ergebniswerte oder Ergebnisse unter<br />

Verwendung von Formeln fremder Herkunft gelten nicht als Lösung.<br />

————————————————————————————————————————<br />

Aufgabe 1 (25 Punkte): Für einen dynamischen Tieftonlautsprecher in nicht eingebautem<br />

Zustand sind gegeben:<br />

Resonanzfrequenz: ......................f = 60 Hz<br />

0<br />

mechanische Kreisgüte .................Q = 3 M<br />

Wicklungswiderstand....................R = 5 Ω<br />

1<br />

Wandlerfaktor................................M = 10 N/A<br />

Membranmasse.............................m = 30 g<br />

M<br />

Membranfläche.............................S = 314 cm 2<br />

a) Bestimmen Sie das Äquivalenzvolumen V . as<br />

b) Bestimmen Sie die Resonanzfrequenz bei Einbau in ein geschlossenes Gehäuse mit der Größe<br />

V = V . B as<br />

c) Bestimmem Sie die resultierende Kreisgüte Q bei Einbau nach b) ohne zusätzliche Bedämpfung<br />

T<br />

und bei Anschluß an einen Verstärker mit dem Innenwiderstankd R = 0 Ω.<br />

G<br />

Aufgabe 2 (25 Punkte): Gegeben ist ein weißes Rauschen mit dem frequenzunabhängigen Intensi<br />

tätsdichtepegel L R = 20 dB sowie ein Sinuston bei f = 1 kHz mit L = 60 dB.<br />

a) Bestimmen Sie den Intensitätspegel des Rauschens bei breitbandiger Messung in einem Band von<br />

20 Hz bis 20 kHz und bei einer Scmalbandmessung im Band 707 Hz bis 1014 Hz.<br />

b) Bestimmen Sie den Gesamtpegel, wenn beide Geräusche gleichzeitig auftreten bei den unter a)<br />

genannten Bedingungen.<br />

c) Wie groß wird der durch das Rauschen verursachte Anzeigefehler für den Schalldruck des Sinustons<br />

(in %) in den beiden Fällen.<br />

Rechnen Sie mit ρ 0 = 1,2 kg/m 3 und c = 340 m/s


Name: ______________________________________________ Matrikelnummer: ___________ - 2 -<br />

————————————————————————————————————————<br />

Fragenteil: SS 1997 A<br />

————————————————————————————————————————<br />

01) Der Schubmodul beschreibt das Verhalten eines Materials bei<br />

± Normal- ± Druck- ± Transversal-<br />

Beanspruchung<br />

————————————————————————————————————————————————<br />

02) Zwei Geräusche haben Spektren, die in einer Frequenzgruppe liegen. Treten beide gleichzeitig auf, so kann<br />

man ihre Lautheit berechnen aus der Summe der<br />

± Lautstärken<br />

± Einzellautheiten<br />

± Intensitäten der einzelnen Geräusche.<br />

————————————————————————————————————————————————<br />

03) Ein Rauschsignal, dessen Intensitätsdichtepegel mit 10 dB je Dekade fällt, nennt man<br />

± weißes Rauschen ± rosa Rauschen ± Alkoholrausch<br />

————————————————————————————————————————————————<br />

04) Mitteltonlautsprecher arbeiten in einem Frequenzbereich<br />

± kR ≈ 1 ± kR > 1 ± kR < 1<br />

————————————————————————————————————————————————<br />

05) Ein dynamischer Tieftonlautsprecher hat eine Resonanzfrequenz f 0 = 45Hz.<br />

Beim Einbau in ein geschlossenes Gehäuse wird seine Resonanzfrequenz<br />

± gleichbleiben ± sinken ± steigen<br />

————————————————————————————————————————————————<br />

06) Die niedrigste Eigenfrequenz in einem beidseitig offenen Rohr tritt auf, wenn die Rohrlänge gleich<br />

± 1/4 ± 1/2 ± 1 Wellenlänge ist.<br />

————————————————————————————————————————————————<br />

07) Das Produkt Kraft mal Schnelle beschreibt eine<br />

± mechanische Impedanz<br />

± mechanische Leistung<br />

± spezifische Schallfeldimpedanz<br />

————————————————————————————————————————————————<br />

08) Schaltet man zwei gleiche mechanische Federn n in Serie, wird die gesamte Nachgiebigkeit<br />

± n/2 ± 2n ± n 2<br />

————————————————————————————————————————————————<br />

09) In Gasen können folgende Wellenformen auftreten:<br />

± Longitudinalwellen ± Transversalwellen ± Torsionswellen<br />

————————————————————————————————————————————————<br />

10) In einem akustischen Meßrohr, das mit einem Reflexionsfaktor R = 0,8 abgeschlossen ist,<br />

verhalten sich die Maxima der Schalldruckamplituden in der stehenden Welle zu den Minima wie<br />

± 0,8:1 ± 5:1 ± 9:1<br />

————————————————————————————————————————————————<br />

11) Die Schallgeschwindigkeit von Biegewellen in einem schlanken runden Stab<br />

± wächst mit steigendem Stabdurchmesser<br />

± ist proportional zur Stablänge<br />

± hängt nicht von den Abmessungen ab<br />

————————————————————————————————————————————————<br />

12) Schallabsorption bei der Schallausbreitung in Luft beeinflußt die Nachhallzeit eines Raums<br />

± bei hohen Frequenzen<br />

± bei tiefen Frequenzen<br />

± gar nicht<br />

————————————————————————————————————————————————<br />

13) Den Nachhall in einem Raum mit hoher Absorption (α > 0,8) kann man genähert beschreiben durch<br />

± die Nachhallformel von Waetzmann, Schuster und Eyring<br />

± die Nachhallformel von Sabine<br />

± keine der genannten Formeln<br />

————————————————————————————————————————————————


Name: ______________________________________________ Matrikelnummer: ___________ - 3 -<br />

————————————————————————————————————————<br />

Fragenteil: SS 1997 A<br />

————————————————————————————————————————<br />

14) Die Frequenzanalyse eines Geräuschs zeigt<br />

± ein Linienspektrum<br />

± ein kontinuierliches Spektrum<br />

± gar kein Spektrum, weil es nicht periodisch ist<br />

————————————————————————————————————————————————<br />

15) Der Absorptionsgrad an einer Grenzfläche ist definiert als das Verhältnis der Schallenergiedichten<br />

± des hinlaufenden und reflektierten Schalls<br />

± des absorbierten und hinlaufenden Schalls<br />

± des absorbierten und reflektierten Schalls<br />

————————————————————————————————————————————————<br />

16) Luftschall trifft auf eine Grenze zu einem anderen Medium. Dabei wird die gesamte Schallenergie<br />

reflektiert, wenn dieses Medium<br />

± schallweich ± angepaßt ± von beliebiger Impedanz<br />

ist.<br />

————————————————————————————————————————————————<br />

17) In einer Schallwelle mißt man die Schallschnelle v = 5 mm/s. Der Schallpegel beträgt dann<br />

± 60 dB ± 100 dB ± 120 dB<br />

————————————————————————————————————————————————<br />

18) Man vergleiche die Richtcharakteristiken eines elektroakustischen Wandlers beim Betrieb als Sender und<br />

als Empfänger<br />

± die Richtcharakteristik ist ein beiden Fällen gleich<br />

± der Sender zeigt eine schärfere Bündelung<br />

± der Empfänger zeigt eine schärfere Bündelung<br />

————————————————————————————————————————————————<br />

19) Bei einem Schnellemikrofon findet man die "Achtercharakteristik"<br />

± nur bei mittleren Frequenzen<br />

± nur bei sehr hohen Frequenzen<br />

± auch bei sehr tiefen Frequenzen<br />

————————————————————————————————————————————————<br />

20) Die Schwingungsrichtung einer Torsiondwelle in einem schlanken runden Stab ist<br />

± parallel ± senkrecht ± ohne Zusammenhang zur<br />

Ausbreitungsrichtung der Welle<br />

————————————————————————————————————————————————<br />

21) Die Geschwindigkeit der Bewegung eines Punktes in einer mechanischen Schaltung gegenüber<br />

Bezugserde heißt<br />

± Schallgeschwindigkeit ± Beschleunigung ± Schnelle<br />

————————————————————————————————————————————————<br />

22) Ein elektroakustischer Wandler<br />

± setzt elektrische Energie in akustische um<br />

± wandelt Spannungen in Ströme<br />

± transformiert Schnellen an einem Querschnittssprung<br />

————————————————————————————————————————————————<br />

23) Die Knotenregel gilt für<br />

± Schalldrücke in einer akustischen Schaltung<br />

± Schnellen in einer mechanischen Schaltung<br />

± Schallflüsse in einer akustischen Schaltung<br />

————————————————————————————————————————————————<br />

24) Ein Elektret-Kondensatormikrofon benötige zum Betrieb eine Gleichspannung von 5V. Wenn diese<br />

ausfällt, mißt man unmittelbar an den Klemmen der Mikrofonkapsel eine Mikrofonspannung die<br />

± dem Schalldruck proportional ist<br />

± dem Quadrat des Schalldrucks proportional ist<br />

± gleich Null ist<br />

————————————————————————————————————————————————<br />

25) Die an einem mechanischen Widerstand auftretende Kraft ist frequenzunabhängig proportional<br />

± zum Weg ± zur Schnelle ± zur Bechleunigung<br />

————————————————————————————————————————————————


Ergebnisse zu Aufgabe 1 (ohne Lösungsweg):<br />

a) Weg 1: Man berechnet L für xss = 8 mm. Ist L größer oder gleich 100 dB,<br />

ist der Lautsprecher geeignet.<br />

Weg 2: Man bestimmt den nötigen Hub xss für L = 100 dB. Dieser muss<br />

kleiner oder gleich 8 mm sein.<br />

2 b) Weg 1: Pak = veff ! Re Zs,mech =<br />

MÄRZ 02 A1<br />

!xss<br />

2 2<br />

2<br />

! "0c !<br />

kR äq<br />

1+ kR äq<br />

2<br />

2 ! #D2<br />

4<br />

ergibt mit Räq= D/4, c = 345 m/s und ρ0 = 1,2 kg/m 3 Pak = 0,0212 W<br />

im Raum ist , mit wird W/m2 J = 4Pak<br />

2<br />

A A = 16#rh J = 1, 691 ! 10−3 Das ergibt einen Schallpegel L = 10 ! log<br />

= 92,3 dB<br />

10<br />

J<br />

−12W/m 2<br />

Der Lautsprecher kann die Forderung nicht erfüllen.<br />

ZURÜCK


Ergebnisse zu Aufgabe 2 (ohne Lösungsweg):<br />

Biegewellengeschwindigkeit im Stab: cB = Eh2<br />

12✣<br />

ergibt bei f = 74 Hz: cB74 = 26, 07 m/s<br />

und bei f = 222 Hz: cB222 = 45, 155 m/s<br />

MÄRZ 02 A2<br />

1/4<br />

$ 2✜f<br />

Der Laufzeitunterschied ist = 1,621⋅10-3 ✁t = s<br />

x<br />

cB74 − x<br />

cB222<br />

(Hinweis für die Teilnehmer an der Klausur vom 12.3.02: dort wurde mit x = 10 mm gerechnet)<br />

ZURÜCK


Ergebnisse zu Aufgabe 3 (ohne Lösungsweg):<br />

a) Wandlerfaktor: N = mit und<br />

C0U0<br />

C0 = ✒0S<br />

das ergibt: N = 3,707 As/m<br />

d<br />

d<br />

S = ✜D2<br />

4<br />

b) Der Druckübertragungsfaktor wird = 0,88⋅10-3 TpU = V/Pa<br />

U0NS<br />

d<br />

c) Schalldruck bei L = 80 dB: p = 2 $ 10 = 0,2 Pa<br />

−5Pa$10 L/20<br />

Mikrofonspannung: U = p $ TpU = 0,176 mV<br />

(Hinweis für die Teilnehmer an der Klausur vom 12.3.02: dort wurde mit d = 30 µm gerechnet)<br />

MÄRZ 02 A3<br />

ZURÜCK


MÄRZ 02 FR<br />

Antworten zum Fragenteil:<br />

Die Lösung ist: "a" für Antwort 1<br />

"b" für Antwort 2<br />

"c" für Antwort 3<br />

caaba abcab caabc bacbc bcccb<br />

ZURÜCK


Ergebnisse zu Aufgabe 1 (ohne Lösungsweg):<br />

a) Der Laufzeitunterschied ergibt eine Phasenverschiebung zwischen u1 und u2,<br />

was mit dem zusätzlichen Phasenwinkel die Richtwirkung ergibt.<br />

b)<br />

✁x = d cos ✏ ✁✩=−2✜✁x/✘ =− ✜<br />

2 cos ✏ ✩ges = ✜<br />

4 + ✁✩ = ✜<br />

4 (1 − 2cos✏)<br />

ua = u1(1 + e Bezug für :<br />

j✩ ges ) |ua| = |u1| 2+ 2cos✩ges ✄ |u a max| = 2|u 1|<br />

✄ = ua(✏)<br />

ua max = 2+2cos✩ges<br />

2<br />

SEP 01 A1<br />

✩ in Grad<br />

0<br />

20<br />

40<br />

60<br />

80<br />

100<br />

120<br />

140<br />

160<br />

180<br />

mit<br />

✩ges = ✜<br />

4 (1 − 2cos✏)<br />

✄<br />

0,924<br />

0,941<br />

0,978<br />

1,000<br />

0,967<br />

0,863<br />

0,707<br />

0,545<br />

0,426<br />

0,383<br />

u( γ )<br />

2<br />

180<br />

160<br />

200<br />

140<br />

220<br />

120<br />

240<br />

100<br />

0.8<br />

0.6<br />

0.4<br />

0.2<br />

0<br />

80<br />

260 280<br />

γ<br />

60<br />

300<br />

Richtfaktor der Mirkrofonanordnung<br />

40<br />

320<br />

20<br />

340<br />

0<br />

1<br />

0.383


Ergebnisse zu Aufgabe 2 (ohne Lösungsweg):<br />

a) lt. Skript S.54 ist bei fgu der Wert k = g:<br />

SEP 01 A2<br />

fgu = gc<br />

2✜<br />

t g = 2✜fgu<br />

c<br />

= 2✜$2000Hz<br />

345m/s =36, 42 1 m<br />

b) Bei Einbau in eine unendliche Wand ist räq = . Für wird<br />

r2<br />

kräq = 1<br />

räq = und<br />

1<br />

k = 1 g d2 = 2 2 g =<br />

c) lt. Skript S.55 ist l =<br />

ln(R2/R1 )<br />

g<br />

2 2<br />

36,42 m =7, 765cm<br />

= ln(d2/d1 )<br />

g<br />

2<br />

= ln(7,765/3)<br />

36,42<br />

m =2, 61 cm<br />

ZURÜCK


Ergebnisse zu Aufgabe 3 (ohne Lösungsweg):<br />

a) Liegen die Geräusche in derselben Frequenzgruppe, werden die<br />

Intensitäten addiert. Zwei gleiche Intensitäten ergeben einen Anstieg<br />

von 3dB =ˆ 3phon. Also wird Lsges = 73phon<br />

SEP 01 A2<br />

Dazu gehört Sges = 2 0,1 Lsges−40 sone = 9, 85 sone<br />

b) Liegen die Geräusche in unterschiedlichen Frequenzgruppen, werden<br />

die Lautheiten einzeln bestimmt und addiert:<br />

S1 = 2 0,1(70−40) sone = 2 3 sone = 8sone= S 2<br />

t S ges = 16 sone<br />

Dazu gehört Lsges = 40phon + 10 lb S ges =80 phon<br />

(Doppelte Lautheit entspricht einem Lautstärkeanstieg um 10 phon!)<br />

ZURÜCK


SEP 01 FR<br />

Antworten zum Fragenteil:<br />

Die Lösung ist: "a" für Antwort 1<br />

"b" für Antwort 2<br />

"c" für Antwort 3<br />

aabbb bbccc accab bccaa bacca<br />

ZURÜCK


Ergebnisse zu Aufgabe 1 (ohne Lösungsweg)<br />

a) Im Skript findet man: T = BlS<br />

w<br />

hier fehlt noch w: w = ✬0m<br />

Q<br />

Damit wird l = T✬0m<br />

QBS<br />

= 5, 86m<br />

b) Die Übertragungsfunktion findet man in Bild 6.4.4-3.<br />

Dabei gelten die vom Schwingkreis bekannten<br />

Zusammenhänge:<br />

→<br />

Q = f0/✁f f 0 2 = fgufgo fgo = f 0 2 /fgu ✁f = fgo − fgu<br />

→ 2 2 fgu + ✁f $ fgu − f0 = 0 ergibt fgu = 138,6Hz ; fgo = 14138,6Hz<br />

JULI 01 A1<br />

ZURÜCK


Ergebnisse zu Aufgabe 2 (ohne Lösungsweg):<br />

lt. Vorlesung ist J = und<br />

4P A<br />

A = 0, 163 s m<br />

$ V<br />

T<br />

Das ergibt A = 20,54 m 2 , J = 1,363⋅10 -3 W/m 2<br />

und damit den Schallpegel L = 10 lg J J0<br />

JULI 01 A2<br />

= 91, 35dB<br />

ZURÜCK


Ergebnisse zu Aufgabe 3 (ohne Lösungsweg):<br />

a) Reflexionsfaktor R= Z − Z 0<br />

Z + Z 0<br />

R =<br />

3<br />

4 + 9<br />

1 − R<br />

= 0, 832 → m =<br />

1 + R<br />

= 1 + 3j − 1<br />

1 + 3j + 1<br />

= 0, 0197<br />

b) Winkel des Reflexionsfaktors ✩R = 90 ) − arctan 3 2<br />

JULI 01 A3<br />

mit ✩R = 180 wird<br />

) 4 l0<br />

✘ − 1 l0 = ✘ 4<br />

= 3j<br />

2 + 3j<br />

= 33, 69)<br />

✩R<br />

180 ) + 1 = 0, 128m<br />

ZURÜCK


JULI 01 FR<br />

Antworten zum Fragenteil:<br />

Die Lösung ist: "a" für Antwort 1<br />

"b" für Antwort 2<br />

"c" für Antwort 3<br />

cacbb aaaab baccb acbcc bacbc<br />

ZURÜCK


Ergebnisse zu Aufgabe 1 (ohne Lösungsweg):<br />

a) Chassis: nM =<br />

1<br />

= 1, 58 $ 10 Gehäuse:<br />

−3 m<br />

N<br />

✬ 0 2 mM<br />

gesamt: nT = →<br />

nMnB<br />

nM + nB = 6, 61 $ 10−4 m<br />

N<br />

f0T =<br />

nB =<br />

1<br />

2✜ mMnT<br />

V<br />

✯0c 2S 2 = 1, 135 $ 10−3 m N<br />

= 61, 9 HZ<br />

b) VAS ist das Volumen eines Gehäuses mit nB = nM →VAS<br />

= nM ✯0c 2S 2 = 13, 9 Liter<br />

c) Gl. 6.5.1.2-7 wird mit x = f / f 0T (p = j x) :<br />

FEB 01 A1<br />

Tp =<br />

−x 2<br />

1 − x 2 + jx/Q<br />

Die Bedingung Tp(fg) = 1/ 2 führt zu einer quadratischen Gleichung für<br />

y = x2 . Die positive Lösung ist y1 = 0,543 und x1 = y1 = 0,737<br />

Damit wird die untere Grenzfrequenz fgu = 0, 737 f0T = 45, 5 Hz<br />

ZURÜCK


Ergebnisse zu Aufgabe 2 (ohne Lösungsweg):<br />

Die Intensitäte in jeder Oktave ist nach Gl. 4.2.3-2: J = 4P A<br />

Hier müssen die Oktavleistungen berechnet und addiert werden:<br />

P = JA<br />

4 = J0 $ 10L/10 $ 0, 163 s m $ V<br />

4T = 3, 36 $ 10−12 Ws $ 10L/10<br />

T<br />

fm/Hz<br />

250<br />

500<br />

1000<br />

2000<br />

4000<br />

8000<br />

P/✙W<br />

9.57<br />

516,40<br />

53,30<br />

191,30<br />

6,70<br />

1,80<br />

Damit wird die Gesamtleistung:<br />

Pges = 779 ✙W<br />

FEB 01 A2 ZURÜCK


Antworten zum Fragenteil:<br />

Die Lösung ist: "a" für Antwort 1<br />

"b" für Antwort 2<br />

"c" für Antwort 3<br />

abbba bcbcc abcbc aacab baccc<br />

FEB 01 FR ZURÜCK


Ergebnisse zu Aufgabe 1 (ohne Lösungsweg):<br />

Die Wirkleistung an einer mech. Impedanz ist .<br />

Pw = i v 2 Re Z mech<br />

Hier ist für Zmech die Strahlungsimpedanz einzusetzen.<br />

i<br />

v = ✬ ^ x<br />

;<br />

2 = 1, 6 m/s Re Z mech = ✯0cS k2R äq<br />

1+k2R 2 = 2,84 $ 10<br />

äq<br />

−3Ns/m mit Räq = D/4<br />

SEP 00 A1<br />

Damit erhält man Pw = 7,82⋅10 -3 W<br />

2<br />

ZURÜCK


Ergebnisse zu Aufgabe 2 (ohne Lösungsweg):<br />

w1 und w2 werden zusammengefasst zu w = 1 Ns/m<br />

Die mech. Impedanz ist: Zmech = j✬m +<br />

umgeformt zu Zmech =<br />

1<br />

j✬n + 1/w<br />

w<br />

✬nw2<br />

2 + j j✬m −<br />

1 + (✬nw) 1 + (✬nw) 2<br />

Der Imaginärteil wird null für f = 0Hz und f = 300<br />

2✜s<br />

SEP 00 A2<br />

= 47, 75Hz<br />

ZURÜCK


Antworten zum Fragenteil:<br />

Die Lösung ist: "a" für Antwort 1<br />

"b" für Antwort 2<br />

"c" für Antwort 3<br />

abccb cccab ababc bacab aaabc<br />

SEP 00 FR ZURÜCK


Ergebnisse zu Aufgabe 1 (ohne Lösungsweg)<br />

Lösung nach der Spiegelquellenmethode:<br />

JULI 00 A1<br />

r 1<br />

r d<br />

Abstände aus der Geometrie:<br />

rd = 2,06m r1 = 3,20m<br />

inkohärente Signale,<br />

die Intensitäten werden addiert:<br />

J = Pak<br />

4✜r 2<br />

Jges = 1,05 µW/m 2<br />

→ Jd = 0,75 µW/m 2<br />

J1 = 0,298 µW/m 2<br />

Lges = 10 lg J/J0 = 60,2 dB<br />

pges = 10 L/20 = 2⋅10 -2 Pa<br />

ZURÜCK


Ergebnisse zu Aufgabe 2 (ohne Lösungsweg):<br />

a) Da c i ✬ ist, ist der hochfrequenzte Anteil bei 2 kHz<br />

schneller.<br />

b) Biegewellengeschwindigkeit:<br />

JULI 00 A2<br />

cB = 4 Eh2<br />

12✯<br />

$ ✬ = 3, 834m s $ ✬<br />

c B1000 = 304 m/s c B2000 = 430 m/s<br />

✁t =<br />

x<br />

cB1000 − x<br />

cB2000<br />

= 964 µs<br />

ZURÜCK


Ergebnisse zu Aufgabe 3 (ohne Lösungsweg):<br />

Lt. Skript ist: Top = →<br />

U0 $ n $ S<br />

d0<br />

U0 = Top $ d0<br />

n $ S<br />

Membranfläche: S = ✜ r 2 = 7,85 $ 10 −5 m 2<br />

Volumen hinter der Membran V = Sd0 = 3,93 $ 10 −9 m 3<br />

Nachgiebigkeit des Volumens n =<br />

V<br />

✯0c 2S2 = d0<br />

✯0c 2S = 4, 46 $ 10−6 m<br />

N<br />

wenn man alles einsetzt, wird U0 = Top✯0c 2 = 142, 8V<br />

JULI 00 A3<br />

ZURÜCK


Antworten zum Fragenteil:<br />

JULI 00<br />

Die Lösung ist: "a" für Antwort 1<br />

"b" für Antwort 2<br />

"c" für Antwort 3<br />

abbbc bcacc babab cccaa abbca<br />

ZURÜCK


Ergebnisse zu Aufgabe 1 (ohne Lösungsweg)<br />

FEB 00 A1<br />

Im Skript findet man: T = BlS<br />

w<br />

hier fehlt noch w: w = ✬0m<br />

Q<br />

Damit wird l = T✬0m<br />

QBS<br />

= 6, 238m<br />

ZURÜCK


Ergebnisse zu Aufgabe 2 (ohne Lösungsweg):<br />

a) Die Impedanz ist: Z = 1<br />

j✬n +<br />

b) Die Kennfrequenz ist f0 =<br />

1<br />

2✜ n<br />

1<br />

w +<br />

1<br />

1<br />

w + j✬m<br />

= 159Hz<br />

c) Bauelemente der Ersatzschaltung: L = M 2n = 1mH<br />

C = m/M 2 = 1mF<br />

R = M 2 R2 /w = 1✡<br />

FEB 00 A2<br />

R 1<br />

L<br />

C<br />

ZURÜCK


Ergebnisse zu Aufgabe 3 (ohne Lösungsweg):<br />

Die ak. Leistung ist konstant, die Schallintensitäten verhalten<br />

sich umgekehrt proportional zu den Absorptionsflächen.<br />

Aneu = Aalt $ Jalt<br />

Jneu = 0, 163 s m V T $ 10 L 1 −L 2<br />

10 = 30, 453m 2<br />

alter mittl. Absorptionsgrad: ✍1 = Aalt/S = 0, 055<br />

Bilanz: unverkleidete Wand Aa = lh✍1 = 1, 081m 2<br />

Teppichboden AT = 0, 07lb = 2, 555m 2<br />

verkleidete Wände Av = 0, 11(2b + l) = 5, 138m 2<br />

Die Decke muß liefern: AD = Aneu − Aa − AT − Av = 21, 678m 2<br />

also wird: ✍D = AD<br />

lb<br />

FEB 00 A2<br />

= 0, 594<br />

ZURÜCK


Antworten zum Fragenteil:<br />

Die Lösung ist: "a" für Antwort 1<br />

"b" für Antwort 2<br />

"c" für Antwort 3<br />

aacbc caccb abbba bbcac cbaab<br />

ZURÜCK


Ergebnisse zu Aufgabe 1 (ohne Lösungsweg):<br />

a) lt. Vorlesung gilt: ✔ = M2<br />

R1ws<br />

SEP 99 A1<br />

ms<br />

ms + mM<br />

Membranfläche S = 5·10 -3 m 2<br />

äquiv. Radius Räq = 2 cm<br />

Strahlungswiderstand ws = 2,801 kg/s<br />

Mediummasse ms =1,206·10 -4 kg<br />

Damit wird der Wirkungsgrad ✔ = 0,167 %<br />

2<br />

ZURÜCK


Ergebnisse zu Aufgabe 2 (ohne Lösungsweg):<br />

a) Die komplexe Abschlussimpedanz für die einfallende Welle<br />

ist:<br />

Z1 = Z0 + j✬m& = (414 + j5, 655 $ 104) Nsm−3 Damit wird der Reflexionsfaktor R = 0,999786 + j0,014639<br />

und der Absorptiosgrad = 2,143·10-4 ✍<br />

Da die Wand keine Verluste hat, tritt der „absorbierte” Teil<br />

durch. Es wird also<br />

✁L =−10 lg ✍ = 36,7 dB<br />

SEPI 99 A2<br />

ZURÜCK


Antworten zum Fragenteil:<br />

Die Lösung ist: "a" für Antwort 1<br />

"b" für Antwort 2<br />

"c" für Antwort 3<br />

bcbcc bacca caccb cbbba babca<br />

ZURÜCK


Ergebnisse zu Aufgabe 1 (ohne Lösungsweg):<br />

a) w1 und w2 werden<br />

zusammengefasst zu<br />

w = 4 Ns/m<br />

Zmech = j✬m +<br />

1<br />

j✬n + 1/w<br />

1<br />

mn − 1<br />

b) Z wird reell für ✬1 =<br />

f1 n = 154 Hz<br />

2w2 = 968 s−1 c) Z(✬1) =<br />

w<br />

2 = 0,25 Ns/m<br />

1 + (✬1nw)<br />

JUNI 99 A1 ZURÜCK<br />

w<br />

m<br />

n


Ergebnisse zu Aufgabe 2 (ohne Lösungsweg):<br />

a) Die Gesamtlautheit ist die Summe der Lautheiten in<br />

den beiden Frequenzgruppen:<br />

= 16 sone<br />

Sges = S500 + S4000<br />

Daraus berechnet man den Lautstärkepegel:<br />

LSges = 40 phon+<br />

10 lg 16<br />

lg 2<br />

= 80 phon<br />

JUNI 99 A2 ZURÜCK


Ergebnisse zu Aufgabe 3 (ohne Lösungsweg):<br />

a) Die Absorptipnsfläche des Raum ist A = 17m 2<br />

Die Schallintensität bei L=90 dB ist J = 10 -3 Wm -2<br />

Im diffusen Feld braucht man dafür Pak = 4,24•10 -3 W<br />

Dafür wird die elektrische Leistung Pel = 0,606 W<br />

benötigt.<br />

JUNI 99 A3<br />

ZURÜCK


Antworten zum Fragenteil:<br />

Die Lösung ist: "a" für Antwort 1<br />

"b" für Antwort 2<br />

"c" für Antwort 3<br />

cacca cabba bbcbb bcaba caaab<br />

ZURÜCK


Ergebnisse zu Aufgabe 1 (ohne Lösungsweg):<br />

a) Die Schnellen verhalten sich wie die mech. Leitwerte:<br />

H = Yn / Yges mit Yges = und<br />

1<br />

+<br />

1<br />

Yn =<br />

0db<br />

j✬m<br />

Eine mögliche Form für H ist:<br />

H=<br />

-20dB<br />

20lg|H|<br />

j f<br />

f 0<br />

1+jQ f<br />

f −<br />

0 f0 f<br />

1 10 f/f 0<br />

mit b)<br />

c)<br />

w+ 1<br />

j✬n<br />

f0 =<br />

1<br />

2✜ mn<br />

= w<br />

2✜m<br />

1<br />

w+ 1<br />

j✬n<br />

= 8Hz<br />

und Q = m/n /w = 1<br />

f ^ f0 : H|e f<br />

f0<br />

2<br />

f p f0 :|H| e 1<br />

d) |H f/(f0)| = 1<br />

FEB 99 A1 ZURÜCK


Ergebnisse zu Aufgabe 2 (ohne Lösungsweg):<br />

a) Die Oberfläche des Raums ist: S = 242 m2 A = ✍ $ S J= 4P/A k= 10−✁L/10 JA = kJ<br />

Die ✁L-<br />

Werte findet man inder Tabelle auf Seite 94 des<br />

Skripts.<br />

f<br />

A<br />

J<br />

k<br />

JA<br />

1,582•10-3 W/m2 3,3•10<br />

0,48<br />

-3 W/m2 12,1 m2 500 Hz<br />

JAges = JA500 + JA1000 = 5, 124W/m 2<br />

LAges = 10 lg JAges<br />

J0<br />

=97, 1dB<br />

3,542•10-3 W/m2 3,542•10<br />

1<br />

-3 W/m2 16,94 m2 1.000 Hz<br />

FEB 99 A2<br />

ZURÜCK


Antworten zum Fragenteil:<br />

Die Lösung ist: "a" für Antwort 1<br />

"b" für Antwort 2<br />

"c" für Antwort 3<br />

abbbc cacca aacca abacb bccaa<br />

ZURÜCK


Ergebnisse zu Aufgabe 1 (ohne Lösungsweg):<br />

a: Wir nennen:<br />

die spez. Abschlußimpedanz Z spez4 = Z 0<br />

die spez. Imped. rechts am Sprung: Z spez3<br />

die spez. Imped. links am Sprung: Z spez2<br />

die spez. Imped. links am Rohrende: Z spez1<br />

Rechts am Querschnittssprung: Z spez3 bleibt unverändert Z 0 .<br />

Links am Querschnittssprung: Die Z spez verhalten sich wie<br />

die Flächen: Z spez2 = 4Z 0 .<br />

Links am Rohrende: Die Transformation über l 1 = λ ändert<br />

Z spez nicht. Wir finden:<br />

Zak1 = Z spez1<br />

S1<br />

= 4ρ0c<br />

0, 08 m 2 =2, 07 ⋅ 105 Ns<br />

m 5<br />

ZURÜCK<br />

OKT 98 A1


Ergebnisse zu Aufgabe 2 (ohne Lösungsweg):<br />

In den Vorlesungsunterlagen findet man für den Leerlauf-<br />

Druckübertragungsfaktor:<br />

Tp = UnS<br />

d<br />

Man benötigt noch die Nachgiebigkeit n,<br />

die man aus den gegebenen mechanischen<br />

Größen zu n = 1,267· 10 -5 m/N berechnen kann.<br />

Damit wird d = 50,7· 10 -6 m = 50,7 µm<br />

ZURÜCK<br />

OKT 1998 A2


Antworten zum Fragenteil:<br />

Die Lösung ist: "a" für Antwort 1<br />

"b" für Antwort 2<br />

"c" für Antwort 3<br />

abaabbcaccbbcbbaaabbabcba<br />

ZURÜCK ZURÜCK<br />

OKT 98 A3


Ergebnisse zu Aufgabe 1 (ohne Lösungsweg):<br />

a: Ersatzbild: N a = 1,76 · 10 -10 m 5 / N<br />

b: Übertragungsfunktion:<br />

mit f 0 = 686 Hz<br />

M a = 305,6 kg/m 4<br />

c: Die Resonanz tritt bei f 0 auf, hier liegt eine Polstelle.<br />

d: Bode-Diagramm<br />

von |H|:<br />

p<br />

q q<br />

1 2<br />

N<br />

∆L<br />

a<br />

0 dB<br />

-20<br />

-40<br />

M<br />

a<br />

0,1 1<br />

H = q2<br />

q1 =<br />

Polstelle<br />

100<br />

1<br />

1 −ω 2 MaNa<br />

f / f 0<br />

10<br />

=<br />

1<br />

1 −(f/f0) 2<br />

ZURÜCK<br />

JUNI 98 A1


Ergebnisse zu Aufgabe 2 (ohne Lösungsweg):<br />

a: Die äquivalente Absorptionsfläche ist A = 9.27 m 2<br />

Man bestimmt bei r = 0,5 m die Energiedichten im<br />

direkten und im diffusen Schallfeld:<br />

Edir = Pak<br />

4 π r 2 c<br />

Damit wird die gesuchte Pegeldifferenz beim Kugelmikrofon<br />

mit γ = 1:<br />

∆L = 10 lg γEdir<br />

=−1, 32 dB<br />

Ediff<br />

Ediff = 4Pak<br />

Ac<br />

und beim Nierenmikrofon mit γ = 3: ∆L = + 3,45 dB<br />

ZURÜCK<br />

JUNI 98 A2


Antworten zum Fragenteil:<br />

Die Lösung ist: "a" für Antwort 1<br />

"b" für Antwort 2<br />

"c" für Antwort 3<br />

baaaacccbbbcbccacabcaacbb<br />

ZURÜCK ZURÜCK<br />

JUNI 98 A3


Ergebnisse zu Aufgabe 1 (ohne Lösungsweg):<br />

a: Der Lautsprecher muß eine akustische Wirkleistung<br />

P Wak = 12,57·10 -3 W abgeben.<br />

Über das Ersatzbild der Belastung eines Kugelstrahlers<br />

wird:<br />

P Bak = P Wak /(kR äq ) = 0,363 W<br />

ZURÜCK<br />

FEB 98 A1


Ergebnisse zu Aufgabe 2 (ohne Lösungsweg):<br />

a: Über die aus der Vorlesung bekannte Gleichung für<br />

den Übertragungsfaktor eines dynamischen Mikrofons<br />

findet man die erforderliche Drahtlänge im Luftspalt<br />

.<br />

l = 4,91 m und damit die Windungszahl<br />

w = 78.<br />

ZURÜCK<br />

FEB 98 A2


Antworten zum Fragenteil:<br />

Die Lösung ist: "a" für Antwort 1<br />

"b" für Antwort 2<br />

"c" für Antwort 3<br />

bbabcacccacbabbcabcbabbab<br />

ZURÜCK ZURÜCK<br />

FEB 98 A3


Ergebnisse zu Aufgabe 1 (ohne Lösungsweg):<br />

a: Die von der Maschine abgegebene akustische Leistung<br />

beträgt:<br />

P ak = 0,0152 W<br />

b: Neue Schallintensität: J 1 = 10 -6 W/m 2<br />

erforderliche Absorptionsfläche: A 1 = 60982 m 2<br />

Absorptionsexponent hierzu: α' 1 = 352<br />

der Absorptionsgrad wird: α 1 = 1-e -352<br />

Die Wände müßten fast alles absorbieren. Entfernt man<br />

sich im freien Schallfeld so weit von der Quelle, wie es die<br />

Raumdiagonale mit 11,4 m zuläßt, erhält man einen Schallpegel<br />

von ca 69 dB. Die geforderte Pegelabsenkung ist<br />

also durch Erhöhung der Wandabsorption nicht möglich.<br />

ZURÜCK<br />

OKT 1997 A2


Ergebnisse zu Aufgabe 2 (ohne Lösungsweg):<br />

a: Die Schallintensitäten werden addiert. Man erhält:<br />

J ges = 2,03 · 10 -4 W/m 2 und L ges = 83,07 dB<br />

b: Die resultierende Lautheit wird: S ges = 21,66 sone<br />

und der resultierende Lautstärkepegel:<br />

L SG = 84,4 phon<br />

ZURÜCK<br />

OKT 97 A1


Antworten zum Fragenteil:<br />

Die Lösung ist: "a" für Antwort 1<br />

"b" für Antwort 2<br />

"c" für Antwort 3<br />

bbaaabcccbaacbaaabbbcbcca<br />

ZURÜCK ZURÜCK<br />

OKT 97


Ergebnisse zu Aufgabe 1 (ohne Lösungsweg):<br />

a: Das Äquivalenzvolumen beträgt<br />

Vas = 3,3 ⋅ 10 −2 m 3 =33 l<br />

b: Die Resonanzfrequenz nach Einbau in das Gehäuse wird<br />

f 0t = 84,85 Hz<br />

c: Die resultierende Kreisgüte wird Q T = 0,673<br />

ZURÜCK<br />

MÄZ 97 A1


Ergebnisse zu Aufgabe 2 (ohne Lösungsweg):<br />

a: Der Intensitätspegel beträgt bei breitbandiger Messung<br />

( Bandbreite ∆f = 19980 Hz) L J1 = 63,0 dB<br />

und bei Schmalbandmessung<br />

(Bandbreite ∆f = 307 Hz) L J2 = 44,9 dB<br />

b: Gesamtpegel breitbandig: L ges1 = 64,76 dB<br />

Gesamtpegel schmalbandig: L ges2 = 60,13 dB<br />

c: Bestimmt werden die Effektivwerte der Schalldrücke.<br />

Schalldrücke: Fehler:<br />

Signal:<br />

Messung breit:<br />

Messung schmal:<br />

p<br />

F1 = 73 %<br />

F2 = 1,5 %<br />

∼ = 0, 02 Pa<br />

p∼ p<br />

ges2 = 0, 0203 Pa<br />

∼ ges1 = 0, 0346 Pa<br />

ZURÜCK<br />

MÄRZ 1997


Antworten zum Fragenteil:<br />

Die Lösung ist: "a" für Antwort 1<br />

"b" für Antwort 2<br />

"c" für Antwort 3<br />

ccbccbbbacaaabbabacbcacab<br />

ZURÜCK ZURÜCK<br />

MÄRZ 97 A3


Schrifteinbettungen:<br />

Verdana<br />

WP Math A: ±±<br />

Optima Optima<br />

Symbol: αβχδε<br />

Symbol prop: αβχδε<br />

Rockwell<br />

Courier new


Anleitung für dieses Dokument:<br />

Schalten Sie die Ansicht "mit Lesezeichen" ein, falls das Dokument<br />

sich nicht in dieser Ansicht öffnet.<br />

Sie sehen dann links von dieser Seite ein Inhaltverzeichnis, mit<br />

dem Sie schnell zu den einzelnen Seiten der Aufgabenstellung<br />

gelangen. Klicken Sie einfach mit der Maus auf den gewünschten<br />

Eintrag.<br />

Um eine kurze Lösung - ohne Lösungsweg - zu sehen, klicken<br />

Sie bitte mit der Maus irgendwo in den Text oder das Bild der<br />

Aufgabenstellung. Ein Klick in die Lösungsseite bringt Sie zur<br />

Aufgabenstellung <strong>zurück</strong>. Notfalls haben Sie ja noch die "Lesezeichen"<br />

auf der linken Seite.<br />

Ein Mausklick in dieses Feld<br />

bringt sie <strong>zurück</strong> zur Startseite.

Hurra! Ihre Datei wurde hochgeladen und ist bereit für die Veröffentlichung.

Erfolgreich gespeichert!

Leider ist etwas schief gelaufen!